Mesokosmos Broadcast Frontpage Graphics

Ajankääntöjä ja kausaalitimantteja (opus 1)

broadcast

Mesokosmoksen podcast-sarja on maailmaa koettelevan korona-kevään aikana saanut vierelleen uudenlaisen mediamuodon. Mesokosmos tarjoaa tästä lähtien tietoa, pohdintaa ja ajattelun virikkeitä myös videoformaatissa uudessa Broadcast-tuotantosarjassa. Videot ovat nauhoitettuja LIVE-taltiointeja, joita kutsumme nimellä Opus eli teos. Opukset tarjoavat eri tavalla toteutetun mahdollisuuden tutustua – aivan liian usein vain marginaaliryhmissä liikkuviin – mielenkiintoisiin ideoihin, tutkimuksiin, ajattelijoihin ja etsijöihin Suomessa.

Mesokosmoksen Broadcast-tuotannon ensimmäisessä jaksossa (https://youtu.be/de4HcCv3K0w) vierailee Mesokosmokselle jo aiemmin tuttu teoreettisen fysiikan tohtori Matti Pitkänen. Otamme käsittelyyn – ei enempää eikä vähempää kuin iki-ihmeellisen aika-käsitteen. Johdattelemme aihetta pienen historiakatsauksen kautta geometriseen ajan muotoiluun graafisen laskimen avulla. Geometrinen aika on matemaattis-fysikaalinen tapa käsitellä aikaa. Geometrinen aika luo pohjan ymmärtää viime vuosisadan yksiä suurimpia uusia ideoita kosmoksesta eli Albert Einsteinin suhteellisuusteorioita.

Mitä ajan suunta tarkoittaa geometrisessa mielessä, entä vastaanotetut ja lähtevät signaalit valokartiossa? Miten valonnopeuden vakioisuus näyttäytyy aika-avaruusdiagrammissa? Voiko ajan suunta vaihtua, samalla myös kausaalisuuden suunta? Voitaisiinko yleisen suhteellisuusteorian tarjoama gravitaation geometrisointia vaivaava energia-ongelma selättää ja miten? Mikä on TGD:ssä usein esiintuotu nolla-energia-ontologia? Entä pienet ja suuret tilafunktioiden reduktiot?

Pitkiksi vierähtävät monituntiset keskustelut Matin kanssa ovat mielenkiintoisia ja opettavaisia. Tälläkin kertaa katsoja ja kuuntelija saa aimoannoksen syvällistä teoriaa ja filosofiaa, ennakkoluulotonta tulkintaa ja rohkeasti esitettyjä viimeisimpiä ideoita, joista löytyy yhtymäkohtia muun muassa elävään tietoisuuden teoriaan, biologiseen kuolemaan, kausaalilakiin eli karmaan ja jopa reinkarnaatioon. Suosittelemme tätä jaksoa nautittavaksi yhdessä kvanttimekaniikkaa käsittelevän podcast-trilogian kanssa (https://mesokosmos.com/tag/matti-pitkanen/). Luvassa on täyden työpäivän kestävä timanttinen matka korkeampiin ajan ulottuvuuksiin.

Linkkejä

7 kommenttia artikkeliin ”Ajankääntöjä ja kausaalitimantteja (opus 1)

  1. Kvantti-kuunnelman Osan 3/3 käsitteistöä ja näitä nauhoja koskien:

    >>>>>>>>>> vapaa tahto on todellista ja vastaa kvanttiteoriassa tilafunktio reduktiota; fysiikassa luonteva systeemien ja niiden alisysteemien muodostama hierarkia vastaa tajuntojen hierarkiaa. >>>>>>>>>>

    Maailmassa ei ole systeemejä ja alisysteemejä erikseen. Fyysisessä mallissa voi olla useiden hiukkasten systeemi S, joka kuvaa montaa hiukkasta mutta se tekee sen päättämällä jättää muun maailman huomiotta. Tällöin S:n on siis jo päätetty olevan systeemi, mutta malli, joka kuvaa useaa hiukkasta kvanttimekaniikassa on tensoritulo (järjestyksellä ei ole väliä) yhden hiukkasen yksittäisistä suureista, ja näiden voidaan sanoa olevan systeemejä mukamas samassa käsityksessä kuin maailmaa tulisi pitää systeeminä. Kun hiukkasella on suure ja vapausaste kvanttitilan muodossa se on tässä ns. alin aste, mutta kvanttimekaniikassa ei voida edes tätä hiukkasta ja sen mahdollisia jäljelle jääviä suureita nostaa miksikään välisysteemiksi ja kokonaisuudeksi, koska ne eivät ole erityisasemassa.

    Tensoritulolla välisysteemejä ei voi syntyä ensin kahden tensorin tuloina, koska nämäkin järjestykset ovat identtisiä keskenään ja jokainen mahdollinen alisysteemi on vain tuo yhden suureen tila. Silloin tällöin hierarkiat äänissä ovat viitanneet siihen, että ns. kausaalitimantin sisällä on kausaalitimantti, mutta jos nämä ovat määritelmiä sille, mitä yhden kvanttiobjektin pitää ottaa ympäristössä huomioon ollakseen tarpeeksi oikeassa, niin on mahdotonta pitää molempia timantteja perusteltuina, sillä isomman ratkaisu/kehitys sisältää tiedon kaikesta mitä pienemmässä on. Joku voi vain laskea asioita pienellä timantilla erikseen, ja saa sen vähemmän informoituja tuloksia kuin mitä pienessä olevien asioiden ratkaisu voi olla isossa ratkaisussa.

    On myös paljon keksittyjä helppouksia määritellä objekti ja sen ympäristö, mistä ensimmäisiä oli pelkkä todennäköisyyskenttä tai stokastinen ympäristö, jossa objektin tilat näyttävät olevan kytkettyjä johonkin, mutta nämä ovat mallissa satunnaisuusfunktioita.

    Click to access 2006.14055.pdf

    valitsee funktioita, jotka aiheuttavat mittausta kuvaavan dekoherenssin. Ja aikakehitelmä on termo-tasapaino-makroskooppinen approksimaatio nimeltä liikeyhtälöiden hierarkia, mutta näissä kuvissa käyrät toimvat usein samoin kuin allaolevassa keskustelussa ensimmäisestä mittautuneesta.

    Tensoritulo tarkoittaa sitä, että kaikilla suureilla on niiden kaikki mahdolliset arvot. Paikan aaltofunktioiden yhdistäminen tensorituloin tarkoittaa, ettei niitä lasketa yhteen. Jos maailmassa ei olisi mahdollista esiintyä joidenkin arvojen toisten muuttujien jonkun arvojen kanssa, niin vain tämä formalismi osoittaisi sen. Tämä on itseasiassa kaikkein tyypillisin seuraus kietoutumisesta ja mittauksesta. Kaikki koherenssi kahden objektin välillä perustuu joissain selityksissä tensoritulossa muodostuneisiin epädiagonaali-alkioihin, mutta tämä on hyvin herkkää kannanvaihdolle, eikä oikein tuota mitään selvää, sen on varmaan enemmän esiinnyttävä yhdellä objektilla sen osittaistilan tasaisena diagonaalina. Silti jos on teoria, joka ei sisällä ensin periaatetta, että kaikki vaikuttaa kaikkeen, ja sitten tutki tätä tekemällä tensoritulon kaikista suureista, johtaa ensimmäisestä syystä vain huonoihin tuloksiin, mutta jälkimmäisessä tapauksessa se ei kuvaa edes kvantti-ilmiöitä kokonaisuudessaan.

    Vapaata tahtoa ei ole todistettu missään. Eikä sitä ole määritelty eikä se silloin liity tilan romahdukseen enempää kuin tilaan joka on ollut romahtaneena niin kauan, että puhutaan jo klassisesta tilasta, mihin jostain syystä kohta halutaan hiljaa hivuten palata ns. pysyvämpänä tietoisuutena.

    >>>>>>>>>> Annetulla tasolla tajunta kokee ”alitajuntansa” mielikuvina – mekin olemme korkeamman tason tajunnan mielikuvia. Jokainen tajunta intentionaalinen agentti ”ali-tajuntojensa” suhteen määrittelemällä reunaehdot niiden toiminnalle. Voi sanoa, että se toimii intentionaalisena agenttina. Intention tarkempi määrittely vaatisi p-adisen ja adelisen fysiikan introdusoimista. >>>>>>>>>>

    Lauseet eivät ole tässä ihan täysiä, ja sitten toistetaan jotain agentista toiseen kertaan. Ääniversiossa (jota haukutaan sitten pian) tajunnan yhtenä osana pidettiin magneettitiloja M jossain kehosta irti. Onko M siis agentti (mutta tässä on kirjoitettu mitä sattuu kokijoista)? Syy tähän on varmaan se, että kehossa ei voi syntyä kuin klassisia tiloja. Mutta jollain tavalla siinä ja sillä syntyy myös mittauksia M:stä tarkoittaen, että keho vuorovaikuttaa M:n kanssa ja kehoon siirtyy M:stä luettu informaatio. Ja M on ainoa kvanttitilojen systeemi ja se missä vähän romahtaa. On oikein pitää mittavälinettä osana ”systeemiä”, mutta tässä ei edellytetä, että kehossa tapahtusi mitään, mikä tätä vaatisi yhtä paljon. Kun on näin erilaiset objektit sanotaan silti, että M kokee kehon romahduksissaan ja keho kokee M:n luettuna tietona. Nämä eivät ole (eikä niin ole koko ajan sanottu) myöskään kahta eri tajuntaa samanaikaisesti vaan yksi tajunta (ja yksi infomaatiobitti), joka vaatii kaiken klassisen ja M.n toiminnot. Mutta on ainakin väärin sanoa, että tämä on tapaus, jossa oltaisiin korkeammalla jossain tilojen hierarkiassa, kun verrataan M:ää ja kehoa.

    Äänessä on myös kuvailtu kehon osuutta tietoisuudesta jonain hyppäyksinä ajatuksista toisiin. Huomaa että tästä yksinään ei voi päätellä, mitä kertoja oikeastaan tarkoittaa kehon järjestelmän fyysisen kuvauksen klassisuudella vs. kvanttimekaniikalla. Valikoin tässä kehon olevan hänestä täysin klassinen, koska hän hyväksyi jonkin klassisen rajan pelkän planckin vakion perusteella. Sanasta sanaan sanoma oli jotakuinkin sellainen että, hyppäykset ideasta toisiin mittaavat maailmassa muualla olevan tilan, ja näin on sanottu tavallisen tietoisuuden aiheen alla. Tätä voisi tarkemmin ilmaista sitten, että mittaus tapahtuu aina ensin, jolloin tulos muuttaa yhden kehosysteemin klassisen suureen jostain arvosta toiseen arvoon. Tämä on ”hyppäys” kehossa, mutta ”kvanttihyppäys” mahdollisesti vain M:ssä. Jos kehon mittauslaitteisto on kovin huono en koskaan pitäisi mahdollisena määrittää M:n tiloja ja dynamiikkaa sen perusteella, mitä siitä luetaan.

    Äänessä oleva tyyppi pitää tosiasiassa M:ää jonain esim. monien tilojen diskreettisuureiden nippuina, joiden vaihtoehtotulokset riittävät kattamaan kaiken, mitä päähän tulee (kuten vaadittaisiin klassisten transistorien muodostamilta porteilta aivoissa). M:ää ei pidetä minkään omana tajuntana ainakaan ääneen tai kovin usein. Näin ei kannata tehdä, koska ei ole mitään näyttöä, että M ilmeisesti kokisi mitään kehosta tulevaa kovin pitkään vaan että sen tieto äskeisestä vuorovaikutuksesta voi vain hävitä diffuusiossa (informaatiota ei häviä, mutta tämä jaettu informaatio kehon ja yhden M-objektin välillä ei koskaan tule käytetyksi uudestaan juuri heidän välillään ja tieto vaatii tuhansia objekteja kerralla ennen kuin se sisältää mitään). Tämä yhteys on hieman toisin kohdissa, joissa sanotaan M:n syntyneen ehkä jatkuvasti kehon prosessien seurauksena, ja jos tätä mietittäisiin enemmän, niin pitäisi ainakin miettiä vielä, onko M kuitenkin täysin deterministinen kuva kehosta, koska mainitut epä-maxwellilaiset superpositiot pätevät varmasti myös klassisille tiloille ennen kuin ne kaikki synkronoituvat keskinäisessä vuorovaikutuksessa. Vielä mietityttää, miksi koherenssi voi kasvaa, jos M-kenttien vuorovaikutusten tiheys voi kasvaa esim. populaation myötä. Nämä asiat riippuvat siitä kytkeytyvätkö M:n tilat toisiin M tiloihin. Sen käsityksen saa sitä, että osa M:n osittaistiloista ovat kuulema jatkuvasti vuorovaikutuksessa johonkin (Zeno), ja jonkin sekavan käsityksen siitä, että nämä ovat jotain pimeää energiaa (ja planckin vakioista seuraavassa)?

    ”Reunaehdot toiminnalle” on tässä fyysikkojen yhteydessä sama kuin, että ylä määrittää alemman Hamiltonin funktion. Se tuskin kuitenkaan tekee näin ilman, että niiden yhteinen H määrittyy myös molempien välisenä vuorovaikutuksena. Jos taas alempi yksinään kokee jonkinlaisen approksimoidun potentiaalivallin tai ”informaatiota vuorovaikutuksesta”, ei M sitten enää varmaan ole se kvanttiobjekti, mistä on puhe.

    En kuullut mitään varsinaista yhteyttä nimenomaan tällaisen M-objektin ja p-adismin jne. välillä, koska minusta kaikki vaiheet TGD-teoriassa olivat mahdollisia reaaliluvuilla (koska toisin ei sanottu ja koska tämä on minusta kohta aika selvää). Teoria on sekin tietysti oikeasti turha M:n kannalta, sillä M:n oli vain todettu olevan hyvin mahdollinen, koska jotain magneettiefektiä on havaittu ja niiden voidaan olettaa olevan monisyisiä. Jostain syystä nämä ovat juuri niitä TGD:ssä päädyttyjä tiloja, joissa ”planckin vakiot kasvaa”, eli ne ovat tietääkseni joissain muissa avaruusulottuvuuksissa, missä sitten on mahdollista, että kvanttiefektit ovat hallitsevia kaikessa. Mitään ulottuvuutta ja siihen liittyvää ei ole todistettu.

    Ainoa syy miksi edes kirjoitan, on että sanot p-adisin ja adelismin olevan jotain, mikä on totta ja fysiikkaa kovassa merkityksessä. Ei ole väliä, mikä luku on p-adinen, kunhan on. Ja päämäärä ”intention tarkempi määrittely” on niin epämääräistä että se kuulostaa kuin pitäisit sitä jonain erillisenä tajuntana taas, jonka olemassaolo perustuisi johonkin seuraavaan. Olisit voinut tarkoittaa että meilläpä on intention ratkaisu olemassa ja teimme sen tällä esityksellä, mutta eipä siltä kuulostanut.

    Jos aaltofuntiot esitetään p-adisina, tämä ei muuta kvanttifysiikkaa lainkaan sen suhteen, mitä se tarkoittaa ja mitä sillä voi kuvata. Aiheesta on olemassa tietoa enemmän kysymällä myös, miksi aaltofuntio on kompleksiluku. Tätä eivät kaikki lähteet käsittele sillä kritiikillä, mitä haen takaa, mutta ainakin löytyy paljon esityksiä, joissa mikään ei oleellisesti muutu. Ks. ainakin ensimmäinen vastaus:
    https://physics.stackexchange.com/questions/32422/qm-without-complex-numbers/61282

    Eli jos tilat ovat yleisessä muodossa unitaarimatriiseja, jonka rakenne on orthogonaalisen ja symplektisen ryhmän leikkaus (jotka saattavat yksinään vastata joitain periaatteita, joilla kvanttisysteemit toimivat), tällöin sen symmetriaryhmä sisältää myös koko kompleksisen lineaariryhmän rakenteen, joka kompleksikunta yleensä on hyvä valinta matriisin alkioiksi, mutta voidaan toteuttaa varmaan kasvaneelle joukolle reaalimuuttujia. Kunhan teoria sisältää komponentin jokaista fyysisen suuren arvoa kohden ja sen todennäköisyyden, sekä evoluution, joka on havaittu, ja joka voi perustua useampaan kuin yhteen funktioon per suure, niin kyseessä on kvanttimekaniikan malli. Kun kompleksiluvut keksittiin, niillä korvattiin reaalilukujen algebra ratkaisemaan rotaatioita, eikä helppouden lisäksi usein ole merkitystä sillä, että uusi algebra on suljettu. Rotaatioihin palautuvat paulin-matriisi -tilat ja ainakin Diracin yhtälö on kirjoitettu muotoon, joka voi käyttää uusia reaalimatriiseja:

    Click to access ImagNumbersArentReal.pdf

    P-adisista luvuista on vaikea keneltäkään saada selvää, onko niiden tarkoitus tehdä avaruudesta diskreetti, jos ne edes on käytetty konfiguraatioavaruudessa tai aaltofunkion alkioiden muuttujien korvaamiseen, vai pidetäänkö teoriaa niiden kanssa edelleen jatkuvana. Jälkimmäinen on mahdollista tekemällä surjektio-kuvaus reaalikunnasta p-adiciin, ja surjektio takasin reaaliin. Tällöin tapaus on sama kuin reaali ja kompleksitilojen välillä paitsi ettei dimensio vaihdu. Tuloksen pätevyys rippuu operaattorien, derivoinnin ja integroinnin laadusta. Maailma tulee huomanneeksi, että on tehty surjektio reaaliarvoisesta fysiikasta, eikä intentio ilmene.

    Topologisesti p-adiset luvut ovat kuin reaalisuoran Cantorin-joukko, jossa on rakoja ja johon myös löytää kartalla. Diskreeteissä kvanttiteorioissa, jotka eivät ole p-adisia, voi tapahtua paljon tai vähän uutta, mitä ei muuten olisi, mutta eniten pitkillä etäisyyksillä, kun ei saisi. Muuten ne pyrkivät olemaan (litteää avaruutta varten) samoja kuin kompleksinen ja reaalinen teoria. Jos maailma on diskreetti, joskus tulee idealistisesti teoria, joka on kaikissa asioissa diskreetti ja jonka dynamiikka muuttaa itseään sopivaksi jatkuvan avaruuden hyvien ratkaisujen suuntaan. Tällöin siis ei voitaisi sanoa, että diskreettiys on jotain, mitä vaaditaan intention olemiseksi, vaan ne ehdot, jotka eivät vähennä sen vaikutuksia, missä tämä tilannekin voi jäädä reunaehdon rooliin.

    Äänessä mainitaan usein jotain olevan vialla kahden ajanhetken välillä ja tilan evoluutiossa. Kvanttihyppyä, jolla tässä tarkoitetaan ehkä eniten tilojen purkautumisia atomeissa, jotka mallinnetaan yhdellä hiukkasella Coulombin potentiaalissa, joissa ei pitäisi olla kuin korkeampi tila läsnä todennäköisyydellä 1, ja tämä säilyy aikakehityksessä. Asia ratkaistiin stokastisen kentän lisäksi lisäämällä systeemiin fotonikenttä, ja teoria oli kvanttikenttäteoria, joka lähestyminen ei kelpaa myöskään muista syistä TGD:n tekijälle.

    >>>>>>>>>> TGD:ssä Bohrin radat ja kvanttitila tekevät paluun nollaenergiaontologiassa. Schrödingerin amplitudin vastineena on kvanttisuperpositio Bohrin ratojen analogioista.
    Tällä tavalla ratkeaa kvanttimekaniikan mittausteorian perusongelma koska kvanttihypyt ovat näiden superpositioiden välillä, eivätkä siis riko klassisen fysiikan lakeja muuttamalla epäjatkuvasti yksittäistä rataa.>>>>>>>>>>

    Tämä ei riitä, vaan superposition lisäksi ominaisenergia- ket-tiloista monet ovat väärin stabiileja. TGD:stä kerrotaan ääneen oikeasti, että sen jokin oleellinen superpositio on eri aikoina olevista tiloista, eli vaikka kyseessä olisi vain yksi Bohrin rata ”alussa”. Jos tässä muodostetaan systeemin dynamiikkaa eikä yksittäistä tilaa, niin en osaa verrata tätä siihen, että elektronin pitää emittoida fotoni sen ja kentän kytkennän avulla. TGD:n vuorovaikutukset ovat taas jossain muualla sen faasiavaruus-vastaavuudessa eivätkä eri ajat ole identtisiä? Eri aikojen superpositio kuulostaa siltä, että jos kvanttiobjekti vuorovaikuttaa kaikkien pisteiden kanssa esim. kvanttikentässä (joskus mahdollisen gravitaatiokentän kanssa, tai pelkän avaruuden kentän kuten Bohmin kvanttipotentiaalin kanssa, jollaiset voivat olla joidenkin mielestä vain useita dynaamisia aallon muotoisia funkioita ja silti muodostaa kvanttiobjektien mitattavia tilafunktioita), niin siihen vaikuttaa koko konfiguraatioavaruus ja siksi myös ajan pisteet, joiden yli pitää ottaa tensoritulo tai sen yhtenä jatkuvan avaruuden vastaavuutena polkuintegraali. Idea vuorovaikutuksissa on kuitenkin jonkinlainen vaikutuksen fysikaalinen mitta, määriteltynä infinitesimaaliselle ajalle, joka on objektien sen hetkinen kuva ympäristöstä. Kvanttikenttäteoriaa tai Diracin-yhtälöä ei muuten yleensä kuvailla siten, että niissä on saavutettu (joskus) laajempi käsite kaiken vaikuttamisesta kaikkeen, ja lisätty signaalinopeus, millä kaikki voi vaikuttaa kaikkeen, jotta tämä olisi realistisen relativistista.

    Muita hyppyjä on esim. kaikki mittausta muistuttavat tapahtumat, joissa sekoitetun tilan todennäköisyyksien voi ajatella hyppäävän tulokseen, mutta nämä myös lakkaavat olemasta äärimmäisen nopeita hyppyjä, jos mittaväline lisätään systeemin ja kokonaistila kehittyy äärellisessä ajassa mittaustulokseen.

    >>>>>>>>>> Kvanttibiologinen tulkinta olisi seuraava. Oletetaan efektiivisten Planckin vakioiden heff = n ⨯ h0 hierarkia, joka liitetään valaisemattomaan aineeseen tavallisen aineen faaseina. Valaisematon aine olisi lokalisoitunut magneettiselle keholle ja tekisi bio-aineesta elävän. Koska heff mittaa evoluutiotasoa algebrallisen kompleksisuuden mittana – ”ÄO”:na – saadaan myös evoluutiohierarkia. Intentionaalinen agentti kontrolloi – ”pomottaa” – tajunnan tasoja, joilla on pienempi heff. Itseisorganisaation teoriassa tämä hierarkia vastaa isäntä-orja hierarkiaa. TGD:ssä itseisorganisaatio palautuu valaisemattomaan aineeseen ja nollaenergiaontologiaan (ZEO).>>>>>>>>>>

    Planckin useat vakiot vaikuttavat tässä mallissa, jossa sanotaan ääneen olevan kvantteja kuvattuna avaruudellisin termein ja vuorovaikutuksia avaruuden rakenteessa, siltä kuin kaikki kytkentävakiot olisi imetty h:hon (kytkettyjen muuttujien faasiavaruus pelkkään avaruuteen tai ns. pintaan), ja siitä keskusteleminen käy samoin kuin vuorovaikutuksista keskusteleminen käy alussa. Muista puheista näkyy myös sellaisia hierarkia-käsityksiä, mitkä koskevat ennenkaikkea suurien systeemien kvanttiromahtamisten etenemisessä, ja tämä liitetään moneen muuhun hierarkiatarpeeseen. On ehkä niin, että herkimmin vuorovaikuttavat suuret romahtavat ensimmäisinä, muistuttaen vaimennettua oskillaattoria, jossa ulkopuolinen kytkentävakio määrää amplitudin pienemisen ja on osa oskilloinnin taajuutta. Joskus näihin romahtajiin kietoutuminen määrää muiden objektien tilatodennälöisyyksiä vahvemmin kuin niiden oma vuorovaikutus ulkopuolisen kohteen kanssa. Varsinkin jos jälkimmäistä ei pidä olla, kuten on kokeissa, jotka ovat valmistaneet tilat joksikin, ja eristäneet kietoutuneiden tilojen kehityksen.

    Päinvastaiseen liittyy vielä yksi asia, jota on saatettu haluta pitää valmiin kietoutumisen sijasta suorien vuorovaikutusten hierarkiana, nimittäin siten, että usean hyvin erilaisten kvanttiobjektin läpi muodostuisi jonkinlainen kytkentöjen rengas. Vakioenergisessä suljetussa ja epärelativistisessa tapauksessa nämä olisivat tosin kaikki yhtä herkkiä yhtä aikaa, koska kaikkien osatilat muodostuvat osa-aaltofunktioista ja oskilloinneista, joiden kulmanopeus on additiivinen kytkentävakioiden funktio. Ja eri aaltojen amplitudien suuruus riippuisi alkutilasta.

    Jos mittalaitteet koostuisivat useista objekteista, niiden yhteisvaikutus on dekoheroiva silloin kun hetkellä t voimakkain hiukkanen ja sen vuorovaikutus on vaiheessa hieman edellä edellistä. Tällainen mittausmalli olisi täysin deterministinen myös täysille mittauksille vaikka mikään objekteista ei olisi lokalisoitunut tai jos ne ovat kvanttikenttä. Ensimmäisenä romahtanut tila ei omaa mitään uutta dynamiikkaa vaan seuraavat romahtamiset ovat edelleen samaa kehitystä. Ja heikoille mittauksille olisi paljon tunnettuja lopputiloja laitteen sisällä. Luulen myös ettei h:n kasvatus merkitse paljon systeemien koherenssille, jos se ei kasvata kytkentävakioita ilman, että se tuo useampia objekteja vuorovaikutuksen piiriin, mikä vastaakin klassisen aineen liiallista tiheyttä, kun useaat yhdistetyt tilat eivät olekaan kuin dekoherenssin lähteitä.

    Kun halutaan käsitellä suuria kokonaisuuksia törmättäisiin ainakin kahteen ominaisuuteen. Mittaavat objektit eivät välttämättä lannista yhtään kahden tarkasteltavan objektin välistä vaikutusta lopputilaan. Jälkimmäinen voi teoriassa muodostaa tilatodennäköisyyksiin omia osa-aaltojaan, joissa yksittäiset amplitudit säilyvät. Toisessa tapauksessa samanlainen vaimeneminen tapahtuu kaikissa komponenteissa. Sen lisäksi, että on tapahtunut vaimeneminen, pitäisi kuitenkin huomata, että näissä kummassakin tapauksessa jos aloitettaisiin satunnaisesta alkutilasta, olisi kuitenkin lukuisia (deterministisiä) arvoja joihin romahdus päätyy. Esimerkit dissipaatio- jne. romahduksista suosivat huom. alinta energiatilaa, ja jotkut toimenpiteet varmasti ylempiä. Hetkelliset ulkopuoliset (resonointi- ja potentiaali-) kentät (kuten alla) johtavat romahdukseen mihin tahansa tilaan, kun kenttä poistuu, mutta voivat siis diffusoida aiemmin stabiilin ja romahtaneen tilan, eivätkä täysin ehkä vastaa ensimmäistä ja toistuvaa projektio-mittausta. Sekoitetut tilat ja heikot mittaukset, mitä on mainittu, eivät ole hyviä määritelmiä keskenään, mutta varmuudella jos lopputila on sekoitettu ja jokin ympäristön tai toisen objektin suure on klassisessa mielessä määrätty prosessin aikana niin se on mitannut ensimmäistä objektia tässä dekoherenssissa heikosti. Taas jos mitattavuusherkkyys harkitsisi sitä, mistä mittaustulos eniten johtuu, niin keskeisvuorovaikutukset olisivat yhtä tärkeitä. Näissä puolestaan ei ole usein eroa kahden erisuuruisen kytkentävakion ja erilaisten populaatioiden välillä, koska se vaikuttaa jollain alkutilan määräämällä amplitudilla pelkkään taajuuteen. Vertaa myös ideaan populaatioden mittaamasta objektista yllä, jossa jos objekteja onkin paljon, niin niistä ensimmäinen romahtaja on todennäköisesti jokin jonka luona esiintyvät resonannssit kyseisen sekoitetun populaation kesken ovat destruktiivisia.

    Tapaus, missä moni tarkasteltava objekti on ehkä suoraan mutta aina välillisesti yhteydessä häviävään resonointikenttään, sisältää myös mahdollisuuksia, joissa vain yksi heikoimmista systeemeistä on alkutilassa ja sellaisessa keskeisvaikutusten vaiheessa, että sen on mahdollista romahtaa puhtaaseen tilaan ja olla kuin kokonaan mitattu osa, kun toiset voivat olla superpositiossa ja jotkut jääneet resonoimaan. Objekteista minkään on hyvin vaikea jäädä niiden resonaation ulkopuolelle, mutta tähän voi päteä jokin ajatus eri vaiheissa olevien aaltojen toistensa kumoamisesta.

    Häviävää kenttää ja erillisiä vuorovaikutuksia varten kirjoitin tiheysmatriisin kahdelle spin1/2-hiukkaselle, joista vain toiseen vaikuttaa x-suunnassa ajastariippuva magnettikenttä be^Bt ja spinien välillä on silti yhtäaikaa Helmholtz-kytkentä vakiolla a. Diracin aikakehityskuvassa pidän kokonaisaikaoperaattoria näiden osien matriisieksponenttien tulona ja aikakehittyvän Hamiltonin funktion osa on kolmannen kertaluvun Dysonin-kehitelmä, joka suppenee, kun B >>>>>>>>> TGD:ssä tajunnalla olisi kaksi komponenttia: a) muuttumaton komponentti, joka vastaisi yleistetyn Zeno-efektin ”kattila-ei-kiehu” komponenttia ja b) muuttuva komponentti, joka vastaisi aistisyötettä ja sen tuottamia ajatuksia ja assosiaatioita: kvanttifysikaalisesti ne vastaisivat nk. ”heikkoja” (weak) kvanttimittauksia. Pysyvä Itse ja Maya ovat itäisten filosofioiden vastineita tälle.>>>>>>>>>>

    Tätä on sanottu ääneen, mutta vaikka tämä tässä muka liittyy TGD:hen, ei tässä ole mitään järjestystä käytössä, ja seuraukset eli mittausvuorovaikutusten laatu perustuu johonkin muuhun kuin eri planck-ulottuvuuksiin? Ylläoleva kyseenalaistaa minkään ominaisuuden, mitä tässäkin on, olevan mahdollista muuten kuin satunnaiselle osalle esim. isoimman h:n populaatiosta kerralla, minkä jälkeen saman objektin odotettu käyttätyminen seuraavassa mittauksessa on taas eri.

    Zeno-efekti on vahvin kaikessa klassisessa aineessa ja vastaa yhtä hyvin ”pienempää planckia” tiheästi täytetyssä aineessa, eli tämä pysyvä itse on jossain sarjan hännillä. Täyden mittauksen suorittamiseksi vuorovaikutuksen kytkentätermit on koettava suurina kohteena olevassa tilassa ja mahdollisimman vähäisenä muualla. Tämän lisäksi Zeno voi vaatia hyvin tiheästi toistuvia vuorovaikutuksia. Kun sanon tämän olevan jotenkin klassista tarkoitan, että tässä täytyy lähestyä ideaalista rajaa, joka ilman populaatiota vaatii silti melkein äärettömästi energiaa tai painetta, joka estää vuorovaikuttavia kohteita hajaantumasta. Zeno-efekti pitkällä aikavälillä tai pieniä virheitä sisältäen on kuitenkin kaikkein vähiten klassisesti käyttäytyvä systeemi, koska jossain vaiheessa jokin vähän 1:stä poikkeava tilan todennäköisyys toteutuu ja tämän myötä muuttuvat samalla kietoutuneiden tilojen osittaistilojen alkiot, jotka oskilloivassa aikakehityksessä ovat todennäköisesti olleet jakautuneet tasan.

    Jos ajatuksella ei ole määritelmää on kai helppo sanoa, että se on vaikka heikko mittaus. Jos ajatus on jotain samanlaista kuin yllä on kirjoitettu klassisesta muuttujasta, niin jos se pitääkin muuttaa heikon mittauksen perusteella, niin tämä tarkoittaa vain ettei ajatuksen muuttumisella ole paljon yhteyttä magneettikehon M kanssa, se pysyy todennäköisemmin vakiona, jos se on omillaan aina vakio. Heikot mittaukset ja zeno eivät silti niin paljon liity aiheeseen muuttumattomista tai muuttuvista tiloista, että kyseessä olisi ihan vastaavuus, ja muuttumattomuus on näiden molempien lähin aihe.

    Tykkää

    1. Iso kiitos isosta kommentista ja kontribuutiosta aiheeseen! Seuraavassa Matin vastine dialogiin:

      [K]=Kommentoija, [MP]=Matti Pitkänen

      >>>>>>>>>> vapaa tahto on todellista ja vastaa kvanttiteoriassa tilafunktio reduktiota; fysiikassa luonteva systeemien ja niiden alisysteemien muodostama hierarkia vastaa tajuntojen hierarkiaa. >>>>>>>>>>

      [K] Maailmassa ei ole systeemejä ja alisysteemejä erikseen. Fyysisessä mallissa voi olla useiden hiukkasten systeemi S, joka kuvaa montaa hiukkasta mutta se tekee sen päättämällä jättää muun maailman huomiotta. Tällöin S:n on siis jo päätetty olevan systeemi, mutta malli, joka kuvaa useaa hiukkasta kvanttimekaniikassa on tensoritulo (järjestyksellä ei ole väliä) yhden hiukkasen yksittäisistä suureista, ja näiden voidaan sanoa olevan systeemejä mukamas samassa käsityksessä kuin maailmaa tulisi pitää systeeminä. Kun hiukkasella on suure ja vapausaste kvanttitilan muodossa se on tässä ns. alin aste, mutta kvanttimekaniikassa ei voida edes tätä hiukkasta ja sen mahdollisia jäljelle jääviä suureita nostaa miksikään välisysteemiksi ja kokonaisuudeksi, koska ne eivät ole erityisasemassa.

      [MP] Systeemin jako kahdeksi erilliseksi kvanttikietoutumattomaksi ali-systeemiksi on tarkalleen se mitä tapahtuu tilafunktion reduktiossa joka taas on tietoisen kokemuksen perus-elementti.

      Mikä tahansa Hilbert avaruus jonka dimensio ei ole alkuluku voidaan jakaa tensorituloksi. Alku-lukudimensioiset Hilbert avaruudet ovat itse asiassa erityisen mielenkiintoisia, koska ne ovat tavallaan jakamattomia alkeis-systeemejä.

      [K] On myös paljon keksittyjä helppouksia määritellä objekti ja sen ympäristö, mistä ensimmäisiä oli pelkkä todennäköisyyskenttä tai stokastinen ympäristö, jossa objektin tilat näyttävät olevan kytkettyjä johonkin, mutta nämä ovat mallissa satunnaisuusfunktioita.

      Click to access 2006.14055.pdf

      valitsee funktioita, jotka aiheuttavat mittausta kuvaavan dekoherenssin. Ja aikakehitelmä on termo-tasapaino-makroskooppinen approksimaatio nimeltä liikeyhtälöiden hierarkia, mutta näissä kuvissa käyrät toimvat usein samoin kuin allaolevassa keskustelussa ensimmäisestä mittautuneesta.

      [MP] Näyttäisi siltä, että kirjoittaja uskoo aaltofunktion dekoherenssin käsitteeseen joka on yksi yritys tulkitsemalla pelastaa kvanttimittausteoria. Näitä pelastusyrityksiä on valtaisa määrä ja näyttää vahvasti siltä että potilas on kuollut ajat sitten.

      Dekoherenssimallissa ei millään tavalla spesifioida miten dekoherenssi tapahtuu. TGD:ssä lähdetään nimenomaan siitä että kvanttimittaukset voivat tapahtua mille tahansa alisysteemille jakaa sen kvanttikietoutumattomaksi tensorituloksi. Universaali observaabeli on tiheysmatriisi.

      [K]Tensoritulo tarkoittaa sitä, että kaikilla suureilla on niiden kaikki mahdolliset arvot. Paikan aaltofunktioiden yhdistäminen tensorituloin tarkoittaa, ettei niitä lasketa yhteen. Jos maailmassa ei olisi mahdollista esiintyä joidenkin arvojen toisten muuttujien jonkun arvojen kanssa, niin vain tämä formalismi osoittaisi sen. Tämä on itseasiassa kaikkein tyypillisin seuraus kietoutumisesta ja mittauksesta.

      [MP] Kaikilla suureilla on kaikki mahdolliset arvot: kaipa tämä voi jotenkin assosioida tensorituloon. Itse lähden matemaattisesta määritelmästä.

      [K] Kaikki koherenssi kahden objektin välillä perustuu joissain selityksissä tensoritulossa muodostuneisiin epädiagonaali-alkioihin, mutta tämä on hyvin herkkää kannanvaihdolle, eikä oikein tuota mitään selvää, sen on varmaan enemmän esiinnyttävä yhdellä objektilla sen osittaistilan tasaisena diagonaalina. Silti jos on teoria, joka ei sisällä ensin periaatetta, että kaikki vaikuttaa kaikkeen, ja sitten tutki tätä tekemällä tensoritulon kaikista suureista, johtaa ensimmäisestä syystä vain huonoihin tuloksiin, mutta jälkimmäisessä tapauksessa se ei kuvaa edes kvantti-ilmiöitä kokonaisuudessaan.

      [MP] Tätä väitettä en kykene ymmärtämään. Joka tapauksessa tensoritulossa kvanttikietoutuminen on se mikä tuottaa kahdesta systeemistä suuremman kvanttikoherentin systeemin.

      [K] Vapaata tahtoa ei ole todistettu missään. Eikä sitä ole määritelty eikä se silloin liity tilan romahdukseen enempää kuin tilaan joka on ollut romahtaneena niin kauan, että puhutaan jo klassisesta tilasta, mihin jostain syystä kohta halutaan hiljaa hivuten palata ns. pysyvämpänä tietoisuutena.

      [MP] Vapaan tahdon olemassaoloa ei varmaan tullakaan koskaan todistamaan matemaattisesti. Kuten ei mitään muutakaan fysiikassakaan oletettua. Koemme vapaan tahdon suoraan ja on täysin mahdotonta tehdä selkoa päivän tekemisistä sortumatta käyttämään ilmauksia, jotka kertovat vapaa tahdon käytöstä.

      TGD:n inspiroima tietoisuuden teoria palauttaa vapaan tahdon tilafunktion reduktioon ja samalla ratkaisee kvanttimittausteorian perusongelman.
      Esimerkiksi Minev et al:n tekemät kokeet atomifysiikan tasolla antavat suoraa tukea varsi dramaattisille ennustuksille.
      Samoin Libetin havainnot.

      >>>>>>>>>> Annetulla tasolla tajunta kokee ”alitajuntansa” mielikuvina – mekin olemme korkeamman tason tajunnan mielikuvia. Jokainen tajunta intentionaalinen agentti ”ali-tajuntojensa” suhteen määrittelemällä reunaehdot niiden toiminnalle. Voi sanoa, että se toimii intentionaalisena agenttina. Intention tarkempi määrittely vaatisi p-adisen ja adelisen fysiikan introdusoimista. >>>>>>>>>>

      [K] Lauseet eivät ole tässä ihan täysiä, ja sitten toistetaan jotain agentista toiseen kertaan. Ääniversiossa (jota haukutaan sitten pian) tajunnan yhtenä osana pidettiin magneettitiloja M jossain kehosta irti.

      Onko M siis agentti (mutta tässä on kirjoitettu mitä sattuu kokijoista)? Syy tähän on varmaan se, että kehossa ei voi syntyä kuin klassisia tiloja. Mutta jollain tavalla siinä ja sillä syntyy myös mittauksia M:stä tarkoittaen, että keho vuorovaikuttaa M:n kanssa ja kehoon siirtyy M:stä luettu informaatio. Ja M on ainoa kvanttitilojen systeemi ja se missä vähän romahtaa. On oikein pitää mittavälinettä osana ”systeemiä”, mutta tässä ei edellytetä, että kehossa tapahtusi mitään, mikä tätä vaatisi yhtä paljon. Kun on näin erilaiset objektit sanotaan silti, että M kokee kehon romahduksissaan ja keho kokee M:n luettuna tietona. Nämä eivät ole (eikä niin ole koko ajan sanottu) myöskään kahta eri tajuntaa samanaikaisesti vaan yksi tajunta (ja yksi infomaatiobitti), joka vaatii kaiken klassisen ja M.n toiminnot. Mutta on ainakin väärin sanoa, että tämä on tapaus, jossa oltaisiin korkeammalla jossain tilojen hierarkiassa, kun verrataan M:ää ja kehoa.

      [MP] M:n (tai siihen liittyvän ”itsen”) voi sanoa olevan agentti tai ”pomo” (master) ja biologisen kehon alainen (slave), jota M kontrolloi. Tämä siksi että siihen liittyvä h_eff:n arvo, joka rationaalilukujen laajennuksen dimensiona on algebrallisen kompleksisuuden mitta (universaali ÄO). Siihen liittyy kvanttiherenssskaala joka on tyypillisesti verrannollinen h_eff ja kasvaa kun h_eff kasvaa. Fiksummat kontrolloivat ei ihan yhtä fiksuja. Tavallinen aine, jolle h_eff>h käyttäytyy kuin pimeä/valaisematon aine. Valaisematon aineen kaltainen aine magneettisella keholla olisi olennaista koko kvanttibiologialle.

      M ja keho eivät tietenkään ole erillisiä. Muuten M ei voisi kontrolloida kehoa. M on vain yleistys Maxwellin teorian mukaisesta magneettikentästä ja eihän sekään ole riippumaton kehosta.

      TGD ennustaa tajuntojen hierarkian: tajuntoja tajunnan sisällä sen kokemina mielikuvina. Geometrinen korrelaatti on avaruusaikalehtien skaala-hierarkia. Lukuteoreettinen visio -adelinen fysiikka joka fuusioi reaalilukupohjaisen fysiikan aistikokemuksen korrelaattina ja p-adiset fysiikat kognition korrelaatteina ennustaa sekä h_eff=nh_0 hierarkian valaisemattomalle aineelle että p-adisten pituus-skaalojen hierarkian. Jokaiseen laajennukseen liittyy joukko alkulukuja, ”ramified primes” jotka määrittelevät preferoituja p-adisia skaaloja laajennukselle.

      [K] Äänessä on myös kuvailtu kehon osuutta tietoisuudesta jonain hyppäyksinä ajatuksista toisiin. Huomaa että tästä yksinään ei voi päätellä, mitä kertoja oikeastaan tarkoittaa kehon järjestelmän fyysisen kuvauksen klassisuudella vs. kvanttimekaniikalla. Valikoin tässä kehon olevan hänestä täysin klassinen, koska hän hyväksyi jonkin klassisen rajan pelkän planckin vakion perusteella. Sanasta sanaan sanoma oli jotakuinkin sellainen että, hyppäykset ideasta toisiin mittaavat maailmassa muualla olevan tilan, ja näin on sanottu tavallisen tietoisuuden aiheen alla. Tätä voisi tarkemmin ilmaista sitten, että mittaus tapahtuu aina ensin, jolloin tulos muuttaa yhden kehosysteemin klassisen suureen jostain arvosta toiseen arvoon. Tämä on ”hyppäys” kehossa, mutta ”kvanttihyppäys” mahdollisesti vain M:ssä. Jos kehon mittauslaitteisto on kovin huono en koskaan pitäisi mahdollisena määrittää M:n tiloja ja dynamiikkaa sen perusteella, mitä siitä luetaan.

      [MP] Tässä on taas massiivinen väärinkäsitys kyseessä. Kirjoittaja lähtee omista oletuksistaan joiden mukaan klassinen fysiikka on kvanttifysiikan raja pitkissä pituus-skaaloissa. TGD:ssä (nolla-energia-ontologiassa) tilanne on täysin toinen. Kvanttitilat ovat superpositioita klassisista deterministisistä aikakehityksistä: 3-D aika=vakio snapshot kvanttitilana on korvattu superpositiona 4-D avaruusaikalehdistä kausaalitimantin sisällä.

      Tällä tavalla klassinen teoria- avaruusaikapintojen geometrodynamiikka – ja myös Diracin yhtälön yleistys – ovat eksakti osa teoriaa.Klassinen teoria ei ole todellakaan approksimaatio. Kvanttihypyt tapahtuvat näiden superpositioiden välillä. Klassinen deterministinen kausaalisuus pätee eikä johda ristiriitaan kvanttihypyn ei-deterministisyyden kanssa. Tämä on koko teorian kuningasidea.

      [K] Äänessä oleva tyyppi pitää tosiasiassa M:ää jonain esim. monien tilojen diskreettisuureiden nippuina, joiden vaihtoehtotulokset riittävät kattamaan kaiken, mitä päähän tulee (kuten vaadittaisiin klassisten transistorien muodostamilta porteilta aivoissa). M:ää ei pidetä minkään omana tajuntana ainakaan ääneen tai kovin usein. Näin ei kannata tehdä, koska ei ole mitään näyttöä, että M ilmeisesti kokisi mitään kehosta tulevaa kovin pitkään vaan että sen tieto äskeisestä vuorovaikutuksesta voi vain hävitä diffuusiossa (informaatiota ei häviä, mutta tämä jaettu informaatio kehon ja yhden M-objektin välillä ei koskaan tule käytetyksi uudestaan juuri heidän välillään ja tieto vaatii tuhansia objekteja kerralla ennen kuin se sisältää mitään).

      Tämä yhteys on hieman toisin kohdissa, joissa sanotaan M:n syntyneen ehkä jatkuvasti kehon prosessien seurauksena, ja jos tätä mietittäisiin enemmän, niin pitäisi ainakin miettiä vielä, onko M kuitenkin täysin deterministinen kuva kehosta, koska mainitut epä-maxwellilaiset superpositiot pätevät varmasti myös klassisille tiloille ennen kuin ne kaikki synkronoituvat keskinäisessä vuorovaikutuksessa. Vielä mietityttää, miksi koherenssi voi kasvaa, jos M-kenttien vuorovaikutusten tiheys voi kasvaa esim. populaation myötä. Nämä asiat riippuvat siitä kytkeytyvätkö M:n tilat toisiin M tiloihin. Sen käsityksen saa sitä, että osa M:n osittaistiloista ovat kuulema jatkuvasti vuorovaikutuksessa johonkin (Zeno), ja jonkin sekavan käsityksen siitä, että nämä ovat jotain pimeää energiaa (ja planckin vakioista seuraavassa)?

      [MP] En voi kuin todeta, että K:n käsitys on todellakin jäänyt erittäin sekavaksi. Ehkä johtuen siitä, että hän on lähtenyt ampumaan norsua hernepyssyllä. Kannattaisi ehkä lukea materiaali kotisivulla, blogissa, kirjoissa ja lukuisissa artikkeleissa. Myös suora keskustelu voisi auttaa: sellaiseen olen aina valmis.

      Kehon dynamiikka ei kiinnitä M:ää täysin. M:n kuitenkin kontrolloi kehon dynamiikkaa muttei täysi.

      Maxwellinkaan teoriassa sm-kentät eivät määrity pelkästään lähteistään. Niissä on mukana myös vapaa osa. Magneettiset vuoputket olisivat vastine vakio-magneettikentille, joilla ei ole virtoja lähteinä. Monopolivuon mahdollisuus (heijastaa CP_2:n topologiaa) tekee mahdolliseksi magneettikentät ilman niitä synnyttäviä virtoja. Nämä monopolivuoputket selittävät esimerkiksi kosmisissa skaaloissa esiintyvät magneettikentät jotka ovat mahdottomia standardikosmologissa koska niitä synnyttävät virrat olisivat satunnaisia.

      [K] ”Reunaehdot toiminnalle” on tässä fyysikkojen yhteydessä sama kuin, että ylä määrittää alemman Hamiltonin funktion. Se tuskin kuitenkaan tekee näin ilman, että niiden yhteinen H määrittyy myös molempien välisenä vuorovaikutuksena. Jos taas alempi yksinään kokee jonkinlaisen approksimoidun potentiaalivallin tai ”informaatiota vuorovaikutuksesta”, ei M sitten enää varmaan ole se kvanttiobjekti, mistä on puhe.

      [MP] Ei-relativistisen kvanttifysiikan kuvailu perustuu Hamiltonin käsitteeseen. Se on ok niin kauan kuin Newtonilainen kuva on hyvä approksimaatio ja varmastinen hyvä fenomeologinen työkalu yksinkertaisten systeemien kuvailuun. Siitä ei ole kuitenkaan apua TGDssä.

      Pistemäinen hiukkanen korvataan 3-pinnalla, jonka rata määrittelee avaruusaikapinnan alueen. Säiemallin 1-D säie korvautuu 3-D pinnalla. Vaati kuutisen vuotta vakuuttautua siitä, että kvanttikenttäteorioiden polkuintegraali ei todellakaan toimi. Lopputulos oli käsite ”klassisten maailmojen maailma” avaruusaikapintojen muodostamana ääretön-D Kaehler geometriana, joka pelkästä olemassaolostaan on uniikki jo luuppiavaruuksien tapauksessa, plus ”nolla-energia-ontologia” joka korvaa Newtonilaisen ontologian. Siinä yleinen koordinaatti-invarianssi pakottaa identifoimaan kvanttitilat superpositioina avaruusaikapinnoista jotka ovat analogisia Bohrin radoille.

      [K] En kuullut mitään varsinaista yhteyttä nimenomaan tällaisen M-objektin ja p-adismin jne. välillä, koska minusta kaikki vaiheet TGD-teoriassa olivat mahdollisia reaaliluvuilla (koska toisin ei sanottu ja koska tämä on minusta kohta aika selvää). Teoria on sekin tietysti oikeasti turha M:n kannalta, sillä M:n oli vain todettu olevan hyvin mahdollinen, koska jotain magneettiefektiä on havaittu ja niiden voidaan olettaa olevan monisyisiä. Jostain syystä nämä ovat juuri niitä TGD:ssä päädyttyjä tiloja, joissa ”planckin vakiot kasvaa”, eli ne ovat tietääkseni joissain muissa avaruusulottuvuuksissa, missä sitten on mahdollista, että kvanttiefektit ovat hallitsevia kaikessa. Mitään ulottuvuutta ja siihen liittyvää ei ole todistettu.

      [MP] p-Adiset lukukunnat tulevat mukaan kognition kuvailussa, ne eivät mitenkään erityisesti liity M:ään. Magneettiselle keholla on tietysti myös p-adisen vastineensa ja se toteuttaa p-adiset analogiat kenttäyhtälöille, jotka ovat täsmälleen saman muotoiset.

      Kognitio on mukana kaikilla tasoilla ja voi sanoa, että p-adinen vastineet esimerkiksi hiukkaseen liittyvälle nelipinnalle antavat sen kognitivisia esityksiä. Nämä ovat yksinkertaistuksia reaalitodellisuudella, mutta jo p-adiset massalaskut, joissa massan neliö olisi p-adisen termodynamiikan antama ja analoginen termiseen energiaan, osoittivat menestyksellää kuinka valtavan ennustuskykyinen p-adinen kognitiivinen esitys on. Syynä ovat lukuteoreettiset ehdot jotka esimerkiksi kvantisoivat lämpötilan vastineen ja pakottavat konformi-invarianssin mikä jo kiinnittää massan neliön odotusarvon lähes täysin. Koska esimerkiksi elektronille p= M_127= 2^127-1 on todella iso niin laskut ovat efektiivisesti eksakteja, koska kehitelmä 1/p:n potensseina konvergoi äärimmäisen nopeasti. Reaalinen fysiikka on sellaista että se sallii kognitiivisen esityksen!

      Magneettiset vuoputket ovat klassisen TGD;n perusratkaisuja kuten myös alkeishiukkasia kuvaat CP_2 tyyppiset ekstremaalit ja säteilykenttiiin liittyvät ”massattomat ekstremaalit”. On äärimmäisen luontevaa johtaa TGD:n ennustukset tutkimalla niitä.

      [K] Ainoa syy miksi edes kirjoitan, on että sanot p-adisin ja adelismin olevan jotain, mikä on totta ja fysiikkaa kovassa merkityksessä. Ei ole väliä, mikä luku on p-adinen, kunhan on.

      [MP] p-Adiset luvut ovat hyvin erilaisia verrattuna reaalilukuihin, koska topologia on aivan erilainen. Sortunetko tässä yleiseen väärinkäsitykseen, että reaalilukua nimitetään p-adiseksi, jos se kehitetään p:n potensseina? Sain aikanaan osaksesi monia sääliviä päänpudisteluja tämän väärinkäsityksen vuoksi.

      Sekä reaaliluvut että p-adiset lukukunnat ovat rationaalilukujen täydentymiä. p-Adisista luvuista useimmat ovat äärettömiä reaalitopologiassa. Rationaaliluvut ovat yhteisiä reaaleille ja p-adisille luvuille. Ja yleisemmin rationaalilukujen laajennus on yhteinen reaaleille ja sen indusoimille rationaalienlaajennuksille. Ne määrittelevät lähes uniikin diskretisoinnin avaruusaikapinnalle pisteinä, joille upotus-avaruus-koordinaatit (M^8 tai H=M^4xCP_2, M^8-H duaalisuus) ovat laajennuksessa. Tulkinta on kognitiivisena esityksenä ja sopusoinnussa sen kanssa että kognitio on aina diskreettiä ja myös äärellistä.

      Kuta isompi laajennus sitä enemmän diskretoinnissa on pisteitä ja algebrallisten lukujen rajalla pisteet ovat tiheä joukko avaruusaikapinnalla. Fysiikka määrittelee itse hierarkian approksimaatioita todellisina fysikaalisina tiloina.

      [K] Ja päämäärä ”intention tarkempi määrittely” on niin epämääräistä että se kuulostaa kuin pitäisit sitä jonain erillisenä tajuntana taas, jonka olemassaolo perustuisi johonkin seuraavaan. Olisit voinut tarkoittaa että meilläpä on intention ratkaisu olemassa ja teimme sen tällä esityksellä, mutta eipä siltä kuulostanut.

      [MP] Puhun p-adisista avaruusaikalehdistä intention ja kognition klassisina avaruusaikakorrelaatteina. p-Adiset differentiaalilyhtälöt eivät ole täysin deterministisiä. Integrointivakiot riippuvat äärellisen monesta pinaari-digitistä ja eivät siis ole vakioita vaikka niiden derivaatta häviääkin. Tämä implikoi klassisen non-eterministisyyden. p-Adinen ratkaisu vastaa kokonaisuudessaan reaaliratkaisua vain jos nämä pseudovakiot ovat aitoja vakioita. Voi sanoa, että tällöin p-adinen intentio on realistinen, se voi vastata reaalista avaruusaikapintaa, aikakehitystä. Esimerkiksi kuviteltu aistihavainto vastaisi melkein aistihavaintoa, jossa syöte tulee magneettiselta keholta eikä ulkoapäin. Syöte ei pääse aistireseptoriin saakka. Samoin kuviteltu motorinen aktiviteetti. p-Adista aruusaikapinnasta mielikuvituksen/intention korrelaattina vain osa realisoituisi.

      [K] Jos aaltofuntiot esitetään p-adisina, tämä ei muuta kvanttifysiikkaa lainkaan sen suhteen, mitä se tarkoittaa ja mitä sillä voi kuvata. Aiheesta on olemassa tietoa enemmän kysymällä myös, miksi aaltofuntio on kompleksiluku. Tätä eivät kaikki lähteet käsittele sillä kritiikillä, mitä haen takaa, mutta ainakin löytyy paljon esityksiä, joissa mikään ei oleellisesti muutu. Ks. ainakin ensimmäinen vastaus:
      https://physics.stackexchange.com/questions/32422/qm-without-complex-numbers/61282

      [MP] Tämä ei pidä paikkaansa. p-Adinen fysiikka on jotain aivan muuta kuin reaalinen johtuen erilaisesta jatkuvuuskäsitteestä. Lukuteoria tuo lisäksi valtavan vahvoja lisäehtoja reaalifysiikallekin kun vaaditaan että p-adinen fysiikka on kognitiivinen esitys reaalifysiikalle. Tämän p-adiset massalaskut osoittivat jo ennenkuin sovelsin p-adisia lukuja tietoisuuden ymmärtämiseen.

      [K] Eli jos tilat ovat yleisessä muodossa unitaarimatriiseja, jonka rakenne on orthogonaalisen ja symplektisen ryhmän leikkaus (jotka saattavat yksinään vastata joitain periaatteita, joilla kvanttisysteemit toimivat), tällöin sen symmetriaryhmä sisältää myös koko kompleksisen lineaariryhmän rakenteen, joka kompleksikunta yleensä on hyvä valinta matriisin alkioiksi, mutta voidaan toteuttaa varmaan kasvaneelle joukolle reaalimuuttujia. Kunhan teoria sisältää komponentin jokaista fyysisen suuren arvoa kohden ja sen todennäköisyyden, sekä evoluution, joka on havaittu, ja joka voi perustua useampaan kuin yhteen funktioon per suure, niin kyseessä on kvanttimekaniikan malli. Kun kompleksiluvut keksittiin, niillä korvattiin reaalilukujen algebra ratkaisemaan rotaatioita, eikä helppouden lisäksi usein ole merkitystä sillä, että uusi algebra on suljettu. Rotaatioihin palautuvat paulin-matriisi -tilat ja ainakin Diracin yhtälö on kirjoitettu muotoon, joka voi käyttää uusia reaalimatriiseja:

      Click to access ImagNumbersArentReal.pdf

      [MP] Kompleksiluvut ovat rationaalinen täydentymä jossa algebralliset luvut vielä täydennetään jatkumoksi. Mitä tulee havaitsemaamme fysiikkaan, emme voi tietenkään olettaa että se vastaisi tätä ideaalista rajaa. On kyse myöskin havaitsijan kuvaamisesta, ei minään ideaalisena muukalaisena vaan osana systeemiä. Kognitiiviset esityksen ja laajennusten hierarkia evoluutiohierarkiana tulevat tällöin mukaan.

      Tällä on aika huikeita seurauksia laskennan kannalta. Tavallinen kvanttikomputaatio Turingin koneen yleistyksen pohjalta perustuu rationaalilukujen rekursiivisiin funktioihin. TGDssä ne korvautuvat rationaalilukujen laajennuksiin liittyvillä vastaavilla funktioilla. Saataisiin kokonainen hierarkia komputationalismeja.

      Aivan äskettäin tuli matemaattinen tulos että kvanttikietoutuminen voi tehostaa kvantti laskentaa niin, että siitä tulee paljon tehokkaampaa kuin tavallinen. Tämä on mahdollista tälle hierarkialle.

      Click to access deeptheorem.pdf

      [K] P-adisista luvuista on vaikea keneltäkään saada selvää, onko niiden tarkoitus tehdä avaruudesta diskreetti, jos ne edes on käytetty konfiguraatioavaruudessa tai aaltofunkion alkioiden muuttujien korvaamiseen, vai pidetäänkö teoriaa niiden kanssa edelleen jatkuvana. Jälkimmäinen on mahdollista tekemällä surjektio-kuvaus reaalikunnasta p-adiciin, ja surjektio takasin reaaliin. Tällöin tapaus on sama kuin reaali ja kompleksitilojen välillä paitsi ettei dimensio vaihdu. Tuloksen pätevyys riippuu operaattorien, derivoinnin ja integroinnin laadusta. Maailma tulee huomanneeksi, että on tehty surjektio reaaliarvoisesta fysiikasta, eikä intentio ilmene.

      [MP] p-Adiset luvut voidaan kuvata reaaliluvuille jatkuvasti. Kanoninen identifikaatio on käyttämäni termi. Reaaliluvuille kuvaus vastakkaiseen suuntaan on kaksi arvoinen vastaten reaaliluvun pinaarikehitelmän ei yksikäsitteisyyttä: desimaaliluvuille se vastaa esityksiä 1=1.0000…= 0.99999..

      [K] Topologisesti p-adiset luvut ovat kuin reaalisuoran Cantorin-joukko, jossa on rakoja ja johon myös löytää kartalla. Diskreeteissä kvanttiteorioissa, jotka eivät ole p-adisia, voi tapahtua paljon tai vähän uutta, mitä ei muuten olisi, mutta eniten pitkillä etäisyyksillä, kun ei saisi. Muuten ne pyrkivät olemaan (litteää avaruutta varten) samoja kuin kompleksinen ja reaalinen teoria. Jos maailma on diskreetti, joskus tulee idealistisesti teoria, joka on kaikissa asioissa diskreetti ja jonka dynamiikka muuttaa itseään sopivaksi jatkuvan avaruuden hyvien ratkaisujen suuntaan. Tällöin siis ei voitaisi sanoa, että diskreettiys on jotain, mitä vaaditaan intention olemiseksi, vaan ne ehdot, jotka eivät vähennä sen vaikutuksia, missä tämä tilannekin voi jäädä reunaehdon rooliin.

      [MP] p-Adiset luvut ovat jatkumoja, mutta topologia on toinen kuin reaalilukujen topologia.

      [K] Äänessä mainitaan usein jotain olevan vialla kahden ajanhetken välillä ja tilan evoluutiossa. Kvanttihyppyä, jolla tässä tarkoitetaan ehkä eniten tilojen purkautumisia atomeissa, jotka mallinnetaan yhdellä hiukkasella Coulombin potentiaalissa, joissa ei pitäisi olla kuin korkeampi tila läsnä todennäköisyydellä 1, ja tämä säilyy aikakehityksessä. Asia ratkaistiin stokastisen kentän lisäksi lisäämällä systeemiin fotonikenttä, ja teoria oli kvanttikenttäteoria, joka lähestyminen ei kelpaa myöskään muista syistä TGD:n tekijälle.

      [MP] Ei ihan näin. Kvanttihyppy on aivan yleisesti tilafunktion reduktion nolla-energiaontologiassa: joka tavallinen kvanttihyppy tai sitten ”weak measurement”. Tavallinen kvanttihyppy ei ole kuitenkaan ihan tavallinen: siihen liittyy ajan suunnan kääntyminen. Tälle Minev et al kokee antavat suoraa evidenssiä. Kuten Libetinkin havainnot.

      Kannattaisi ehkä vierailla kotisivulla tai lukea lähteitä joihin jo viittasin ja selitykseen ylempänä. Reaalifysiikka yleistetään lukuteoreettisesti universaaliksi adeliseksi fysiikaksi. Avaruusaikapinta on kuin kirja jolla on suuri määrä lehtiä. Reaalilehti ja p-adiset lehdet kognitiivisina esityksinä. Niiden määrä on äärellinen jos vain ”ramified primes” vastaavat lehdet ovat mukana.

      Tästä on 80 sivuinen artikkelin Springerin julkaisemassa kirjassa. Se löytyy myös kotisivulta
      http://tgdtheory.fi/public_html/articles/adelephysics.pdf .

      >>>>>>>>>> TGD:ssä Bohrin radat ja kvanttitila tekevät paluun nollaenergiaontologiassa. Schrödingerin amplitudin vastineena on kvanttisuperpositio Bohrin ratojen analogioista. Tällä tavalla ratkeaa kvanttimekaniikan mittausteorian perusongelma koska kvanttihypyt ovat näiden superpositioiden välillä, eivätkä siis riko klassisen fysiikan lakeja muuttamalla epäjatkuvasti yksittäistä rataa.>>>>>>>>>>

      [K] Tämä ei riitä, vaan superposition lisäksi ominaisenergia- ket-tiloista monet ovat väärin stabiileja. TGD:stä kerrotaan ääneen oikeasti, että sen jokin oleellinen superpositio on eri aikoina olevista tiloista, eli vaikka kyseessä olisi vain yksi Bohrin rata ”alussa”.

      [MP] Eri aikoja ei superponoida vaan eri avaruusaikapintoja. Jos kyseessä olisivat Bohrin radat olisi kyseessä superposition niiden alkupisteiden suhteen.

      Jos tässä muodostetaan systeemin dynamiikkaa eikä yksittäistä tilaa, niin en osaa verrata tätä siihen, että elektronin pitää emittoida fotoni sen ja kentän kytkennän avulla. TGD:n vuorovaikutukset ovat taas jossain muualla sen faasiavaruus-vastaavuudessa eivätkä eri ajat ole identtisiä?

      [K] Eri aikojen superpositio

      [MP] Eri avaruusaikapintojen, ei eri aikojen. Nollaenergaontologiassa kvanttitila ei ole aika=vakio snapshot vaan superpositio klassisista deterministisistä aikakehityksistä, preferred extremals, jotka ovat analogisia Bohrin radoille.
      Tämä seuraa yleisestä koordinaatti-invarianssista: annettuun kolmipintaan liittyy uniikki nelipinta, jolla yleiset koordinaattimuunnokset 4-D mielessä vaikuttavat. Se implikoi taas holografian: datat 3-pinnalla kiinnittävät 4-pinnan.
      Itse asiassa holografia osoittautuu vahvemmaksi. 2-D data kiinnittävät 4-pinnan.

      [K] kuulostaa siltä, että jos kvanttiobjekti vuorovaikuttaa kaikkien pisteiden kanssa esim. kvanttikentässä (joskus mahdollisen gravitaatiokentän kanssa, tai pelkän avaruuden kentän kuten Bohmin kvanttipotentiaalin kanssa, jollaiset voivat olla joidenkin mielestä vain useita dynaamisia aallon muotoisia funkioita ja silti muodostaa kvanttiobjektien mitattavia tilafunktioita), niin siihen vaikuttaa koko konfiguraatioavaruus ja siksi myös ajan pisteet, joiden yli pitää ottaa tensoritulo tai sen yhtenä jatkuvan avaruuden vastaavuutena polkuintegraali. Idea vuorovaikutuksissa on kuitenkin jonkinlainen vaikutuksen fysikaalinen mitta, määriteltynä infinitesimaaliselle ajalle, joka on objektien sen hetkinen kuva ympäristöstä. Kvanttikenttäteoriaa tai Diracin-yhtälöä ei muuten yleensä kuvailla siten, että niissä on saavutettu (joskus) laajempi käsite kaiken vaikuttamisesta kaikkeen, ja lisätty signaalinopeus, millä kaikki voi vaikuttaa kaikkeen, jotta tämä olisi realistisen relativistista.

      [MP] Vuorovaikutukset redusoituvat topologisella tasolla 3-pinnan – itse asiassa partonisen 2-pinnan -topologian muutokseen. Voidaan ottaa lähtömielikuvaksi Feynmanin graafi prosessille A–B+C . Tärkeä osa 4-pinnan topologiaa ja geometriaa ovat valonkaltaiset kolmipinnat, joilla metriikan signatuuri muuttuu Eulidisesta (CP_2 esktremaali joka kuvaa alkeishiukkasta) Minkowskiseksi, hiukkasen ulkopuolinen maailma. Nämä Feynmanin graafin viivoja, ja niihin liittyvissä hajoamisvertekseissä A–>B+C , sisääntuleva partoninen kaksipinta ja ulosmenevät partoniset 2 pinnan B ja C liittyvät päitään myöten yhteen. Feynmanin graavin A–>B+C yleistys kun pistemäiset hiukkaset korvataan 2-pinnoilla on kyseessä.

      Näillä pinnoilla indusoidut spinorikentät toteuttavat modifioidun Diracin yhtälön joka määräytyy täysin avaruusaikapinnan määräävästä variaatioperiaatteesta sisäisen konsistenssin perusteella (hermiittisyys). Tuloksena on supersymmetria geometristen ja fermionisten vapausasteiden välillä. Jokaiseen säilyvään varaukseen liittyy super-varaus vastaten spinorikentä ratkaisumoodeja.

      [K] Muita hyppyjä on esim. kaikki mittausta muistuttavat tapahtumat, joissa sekoitetun tilan todennäköisyyksien voi ajatella hyppäävän tulokseen, mutta nämä myös lakkaavat olemasta äärimmäisen nopeita hyppyjä, jos mittaväline lisätään systeemin ja kokonaistila kehittyy äärellisessä ajassa mittaustulokseen.

      [MP] Perusennustus on, että aika pienen tilafunktion reduktion välillä on äärellinen kuten myös kahden ison (”tavallisen”).
      Kahta pientä tilafunktion reduktiota yhdistää äärellisen kestoinen unitaarisen evoluution vastine.

      >>>>>>>>>> Kvanttibiologinen tulkinta olisi seuraava. Oletetaan efektiivisten Planckin vakioiden heff = n ⨯ h0 hierarkia, joka liitetään valaisemattomaan aineeseen tavallisen aineen faaseina. Valaisematon aine olisi lokalisoitunut magneettiselle keholle ja tekisi bio-aineesta elävän. Koska heff mittaa evoluutiotasoa algebrallisen kompleksisuuden mittana – ”ÄO”:na – saadaan myös evoluutiohierarkia. Intentionaalinen agentti kontrolloi – ”pomottaa” – tajunnan tasoja, joilla on pienempi heff. Itseisorganisaation teoriassa tämä hierarkia vastaa isäntä-orja hierarkiaa. TGD:ssä itseisorganisaatio palautuu valaisemattomaan aineeseen ja nollaenergiaontologiaan (ZEO).>>>>>>>>>>

      [K] Planckin useat vakiot vaikuttavat tässä mallissa, jossa sanotaan ääneen olevan kvantteja kuvattuna avaruudellisin termein ja vuorovaikutuksia avaruuden rakenteessa, siltä kuin kaikki kytkentävakiot olisi imetty h:hon (kytkettyjen muuttujien faasiavaruus pelkkään avaruuteen tai ns. pintaan), ja siitä keskusteleminen käy samoin kuin vuorovaikutuksista keskusteleminen käy alussa.

      [MP] Ei ihan näin. Kytkentävakioita ei imetä h:n.

      M^8-H dualisuus merkitsee sitä että avaruusaika voidaan tulkinta pintana M^8:ssa tai H:ssa. M^8 kuvassa avaruusaika määräytyy polynomista rationaalikertoimin ja tämä polynomi määrää algebrallisen laajennuksen ja avaruusaikapinnan tason evoluutiohierarkiassa.

      Algebralliset laajennusten parametrit kuten dimensio n=h_eff/h_0 (h= 6h_0 seuraa empiirisesta argumentista) ja ramified primes ovat tärkeitä ja kytkentävakiot ja niiden diskreetti evoluutio laajennuksen funktiona määräytyisi näistä paramereista. On varmasti muitakin laajennusta kuvaavia lukuteoreettisia parametreja. Sironta-amplitudien lausekkeet yksinkertaistuvat, koska avaaruusaika diskretoituu uniikeiksi kognitiiviseksi esitykseksi. Tässä tulee mukaan lukuteoria, joka on tällä hetkellä voimakkaasti kehittymässä.

      Kähler kytkentävakio alpha_K on fundamentaalinen kytkentäparametri ja sen arvo määräytyisi kvanttikriittisyydestä. Vakuumifunktionaalinen eksponentti on analoginen partitiofunktioon ja alpha_K analoginen lämpötilaan. Kvanttikriittisyys universumille sallii vain diskreetin joukon arvoja alpha_K:lle ja ne määräytyisivät algebrallisesta laajennuksesta.

      Mukana on myös kognition dynamiikka. Galois ryhmä ja sen jakaantuminen inkluusiohierarkiaksi normaalialiryhmiä (extension of extension of …), ”ramified primes”, jotka määräytyvät laajennuksen määräämästä polynomista, on fundamentaalinen. Galois ryhmän ryhmäalgebraan liittyvät diskreetti dynamiikka olisi kognition fysiikkaa ja tilafunktion reduktiokaskadit Galois ryhmän ryhmäalgebrassa vastaisivat ehkä kognitiota analyysinä.

      [K] Luulen myös ettei h:n kasvatus merkitse paljon systeemien koherenssille, jos se ei kasvata kytkentävakioita ilman, että se tuo useampia objekteja vuorovaikutuksen piiriin, mikä vastaakin klassisen aineen liiallista tiheyttä, kun useat yhdistetyt tilat eivät olekaan kuin dekoherenssin lähteitä.

      [MP] Katselemalla kvanttiteorioiden peruskaavoja selviää, että kuta suurempi h tai siis h_eff, sitä suurempi kvanttiskaala.

      Compton aallonpituus on verrannollinen h:n. Vetyatomin ratasäde verrannollinen h^2. Planckin vakioiden hierarkia tekee mahdolliseksi kvanttikoherenssin magneettisella keholla joka taas selittää biosysteeminen koherenssin joka on täysi mysteeri jos biologia on pelkkää kemiaa. Olisimme pelkkiä kemikaalisäkkejä, emme yhtään enempää.

      >>>>>>>>> TGD:ssä tajunnalla olisi kaksi komponenttia: a) muuttumaton komponentti, joka vastaisi yleistetyn Zeno-efektin ”kattila-ei-kiehu” komponenttia ja b) muuttuva komponentti, joka vastaisi aistisyötettä ja sen tuottamia ajatuksia ja assosiaatioita: kvanttifysikaalisesti ne vastaisivat nk. ”heikkoja” (weak) kvanttimittauksia. Pysyvä Itse ja Maya ovat itäisten filosofioiden vastineita tälle.>>>>>>>>>>

      [K] Tätä on sanottu ääneen, mutta vaikka tämä tässä muka liittyy TGD:hen, ei tässä ole mitään järjestystä käytössä, ja seuraukset eli mittausvuorovaikutusten laatu perustuu johonkin muuhun kuin eri planck-ulottuvuuksiin? Ylläoleva kyseenalaistaa minkään ominaisuuden, mitä tässäkin on, olevan mahdollista muuten kuin satunnaiselle osalle esim. isoimman h:n populaatiosta kerralla, minkä jälkeen saman objektin odotettu käyttäytyminen seuraavassa mittauksessa on taas eri.

      [MP] Tämä on nollaenergiaontologiaan perustuvan tietoisuuden teorian ennustus. En lähde liikkeelle mistään itämaisesta filosofiasta, jonka yrittäisin reprodusoida. Tämä kahtiajako vastaa sitä, etttä kvanttihyppyjen sarjassa CD:n passiiviseen reunaan liittyvä tila ei muutu: Zeno efektin analogia. Aktiiviseen reunaan liittyvä tila muuttuu jokaisessa unitaarisen evoluution vastineessa jota seuraa ”heikko” mittaus. Tämä vastaa aistisyötettä. Iso (”tavallinen” kvanttihyppy merkitsee ”itsen” kuolemaa ja reinkarnaatiota vastakkaisessa ajan suunnassa. Nyt aktiivisesta CD:n reunasta tulee passiivinen ja päinvastoin. Tälle antaa suoraa tukea Minev et al havainnot kuten myös Libet.

      [K] Zeno-efekti on vahvin kaikessa klassisessa aineessa ja vastaa yhtä hyvin ”pienempää planckia” tiheästi täytetyssä aineessa, eli tämä pysyvä itse on jossain sarjan hännillä. Täyden mittauksen suorittamiseksi vuorovaikutuksen kytkentätermit on koettava suurina kohteena olevassa tilassa ja mahdollisimman vähäisenä muualla. Tämän lisäksi Zeno voi vaatia hyvin tiheästi toistuvia vuorovaikutuksia. Kun sanon tämän olevan jotenkin klassista tarkoitan, että tässä täytyy lähestyä ideaalista rajaa, joka ilman populaatiota vaatii silti melkein äärettömästi energiaa tai painetta, joka estää vuorovaikuttavia kohteita hajaantumasta. Zeno-efekti pitkällä aikavälillä tai pieniä virheitä sisältäen on kuitenkin kaikkein vähiten klassisesti käyttäytyvä systeemi, koska jossain vaiheessa jokin vähän 1:stä poikkeava tilan todennäköisyys toteutuu ja tämän myötä muuttuvat samalla kietoutuneiden tilojen osittaistilojen alkiot, jotka oskilloivassa aikakehityksessä ovat todennäköisesti olleet jakautuneet tasan.

      [MP] ”Itsen kehitys” on analogia Zeno efektille. Nolla-energiaontologian tilat ovat pareja tavallisista 3-D tiloistan CD:n reunoilla. CD:n passiivisella reunalla tila ei muutu – Zeno efekti. Aktiivisella reunalla, jossa tapahtuu heikkoja mittauksia se muuttuu ja se vastaa ”itsen” aistihavaintoja.

      Tykkää

  2. Teksti on katkennut kohdasta
    ”…aikakehittyvän Hamiltonin funktion osa on kolmannen kertaluvun Dysonin-kehitelmä, joka suppenee, kun B …”
    Tämä on puuttuva osa:

    Häviävää kenttää ja erillisiä vuorovaikutuksia varten kirjoitin tiheysmatriisin kahdelle spin1/2-hiukkaselle, joista vain toiseen vaikuttaa x-suunnassa ajastariippuva magnettikenttä be^Bt ja spinien välillä on silti yhtäaikaa Helmholtz-kytkentä vakiolla a. Diracin aikakehityskuvassa pidän kokonaisaikaoperaattoria näiden osien matriisieksponenttien tulona ja aikakehittyvän Hamiltonin funktion osa on kolmannen kertaluvun Dysonin-kehitelmä, joka suppenee, kun B >>>>>>>>> TGD:ssä tajunnalla olisi kaksi komponenttia…

    Tykkää

  3. Teksti katkeaa aina kohdasta
    ”…aikakehittyvän Hamiltonin funktion osa on kolmannen kertaluvun Dysonin-kehitelmä, joka suppenee, kun B …”
    Tämä on puuttuva osa ilman le-symbolia:

    Häviävää kenttää ja erillisiä vuorovaikutuksia varten kirjoitin tiheysmatriisin kahdelle spin1/2-hiukkaselle, joista vain toiseen vaikuttaa x-suunnassa ajastariippuva magnettikenttä be^Bt ja spinien välillä on silti yhtäaikaa Helmholtz-kytkentä vakiolla a. Diracin aikakehityskuvassa pidän kokonaisaikaoperaattoria näiden osien matriisieksponenttien tulona ja aikakehittyvän Hamiltonin funktion osa on kolmannen kertaluvun Dysonin-kehitelmä, joka suppenee, kun B pienempikuin 0 ja kenttä poistuu. Nämä valinnat voivat määrittää, miten kaksi eri aikakehitelmä oikeasti suhtautuvat toisiinsa, eikä tätä voi pitää kuin häiriö-approksimaationa. Lopputuloksessa a:n suurudella ei esiinny taajuuden lisäksi muuta vaikutusta. Jos alkutilat ovat tilojen -+ ja ++ superpositio, missä edellisen matriisielementti on alussa q ja jälkimmäisen 1-q, nämä samat matriisielementit kehittyvät seuraavanlaisesti

    y=-((b^{6}*q*e^{6*t*B}-b^{6}*e^{6*t*B}-6*b^{6}*q*e^{5*t*B}+6*b^{6}*e^{5*t*B}-21*b^{4}*q*B^{2}*e^{4*t*B}+12*b^{4}*B^{2}*e^{4*t*B}+15*b^{6}*q*e^{4*t*B}-15*b^{6}*e^{4*t*B}+84*b^{4}*q*B^{2}*e^{3*t*B}-48*b^{4}*B^{2}*e^{3*t*B}-20*b^{6}*q*e^{3*t*B}+20*b^{6}*e^{3*t*B}+72*b^{2}*q*B^{4}*e^{2*t*B}-36*b^{2}*B^{4}*e^{2*t*B}-126*b^{4}*q*B^{2}*e^{2*t*B}+72*b^{4}*B^{2}*e^{2*t*B}+15*b^{6}*q*e^{2*t*B}-15*b^{6}*e^{2*t*B}-144*b^{2}*q*B^{4}*e^{t*B}+72*b^{2}*B^{4}*e^{t*B}+84*b^{4}*q*B^{2}*e^{t*B}-48*b^{4}*B^{2}*e^{t*B}-6*b^{6}*q*e^{t*B}+6*b^{6}*e^{t*B}-36*q*B^{6}+72*b^{2}*q*B^{4}-36*b^{2}*B^{4}-21*b^{4}*q*B^{2}+12*b^{4}*B^{2}+b^{6}*q-b^{6})*(\cos4*a*t+1))/(2*(b^{6}*e^{6*t*B}-6*b^{6}*e^{5*t*B}-3*b^{4}*B^{2}*e^{4*t*B}+15*b^{6}*e^{4*t*B}+12*b^{4}*B^{2}*e^{3*t*B}-20*b^{6}*e^{3*t*B}-18*b^{4}*B^{2}*e^{2*t*B}+15*b^{6}*e^{2*t*B}+12*b^{4}*B^{2}*e^{t*B}-6*b^{6}*e^{t*B}+36*B^{6}-3*b^{4}*B^{2}+b^{6}))

    y=(b^{6}*q*e^{6*t*B}-6*b^{6}*q*e^{5*t*B}-21*b^{4}*q*B^{2}*e^{4*t*B}+9*b^{4}*B^{2}*e^{4*t*B}+15*b^{6}*q*e^{4*t*B}+84*b^{4}*q*B^{2}*e^{3*t*B}-36*b^{4}*B^{2}*e^{3*t*B}-20*b^{6}*q*e^{3*t*B}+72*b^{2}*q*B^{4}*e^{2*t*B}-36*b^{2}*B^{4}*e^{2*t*B}-126*b^{4}*q*B^{2}*e^{2*t*B}+54*b^{4}*B^{2}*e^{2*t*B}+15*b^{6}*q*e^{2*t*B}-144*b^{2}*q*B^{4}*e^{t*B}+72*b^{2}*B^{4}*e^{t*B}+84*b^{4}*q*B^{2}*e^{t*B}-36*b^{4}*B^{2}*e^{t*B}-6*b^{6}*q*e^{t*B}-36*q*B^{6}+36*B^{6}+72*b^{2}*q*B^{4}-36*b^{2}*B^{4}-21*b^{4}*q*B^{2}+9*b^{4}*B^{2}+b^{6}*q)/(b^{6}*e^{6*t*B}-6*b^{6}*e^{5*t*B}-3*b^{4}*B^{2}*e^{4*t*B}+15*b^{6}*e^{4*t*B}+12*b^{4}*B^{2}*e^{3*t*B}-20*b^{6}*e^{3*t*B}-18*b^{4}*B^{2}*e^{2*t*B}+15*b^{6}*e^{2*t*B}+12*b^{4}*B^{2}*e^{t*B}-6*b^{6}*e^{t*B}+36*B^{6}-3*b^{4}*B^{2}+b^{6})
    https://www.desmos.com/calculator
    -formaatissa.

    Tilan +- todennäköisyys on sama kuin q(t+v/2), missä v on a:sta riippuva aallonpituus. — lähestyy nollaa kaikkina hetkinä. B on sekä kytkentä, suuruus että keston määrä, mutta b parametrisoi pelkästään ensimmäistä, jos halutaan. Suuret b-arvot johtavat jyrkempiin kehityksiin kohti lopputilaa, joka on saavutettu viimeistään, kun t=10/bB, mutta sisältää eri välejä sadasosien välissä, missä tilat tai ++ vaihtelee lineaarisesti nollasta yhteen ja saattaa vielä muodostaa uuden aallonhuipun b-resonaatiossa. Ero systeemien a=0 ja a!=0 välillä on se, että edelliset päätyvät aina stabiileiksi, mutta muutoin -+ ja +- tilojen on lähestyttävä nollaa taikka niiden resonaatio jää jäljelle käyden aina a=0 -tapauksesta nollaan.

    Huom. että yhdessä äänessä sanotaan ikäänkuin energian kasvavan, samalla kun h:t kasvaa, mutta ennenkuin tulin kertomaan aallonpituuden merkityksestä tällä on haluttu saada aikaiseksi varmaan kerralla kaikkien energian saamien arvojen kasvu (potentiaali ero) stabiileissa tiloissa ja perustuen vain kytkentävakioihin. Näin ei oikeasti kävisi millään energian syömisellä, vaan jos kvanttiobjekti pakotetaan ottamaan energiaa vastaan vuorovaikutuksessa, se joko luhistuu ylemmälle energian aaltofunktiolle tai sen ylempien tilojen todennäköisyys kasvaa, jos enempää tiloja ei ole niin energia menee ohi tai alkaa tapahtua muuta hiukkasfysiikkaa. Jos koko vieraan ulottuvuuden energiatiheys muuttuu, ovat jotkin hiukkaset ehkä parempia kuin toiset ja kytkeytyvät eri tavalla.

    Lopulta jos vuorovaikutus näkyy jossain ensimmäisenä niin kvanttimekaniikassa katseltaisiin silloin alkioita tiheysmatriisissa, joka on ratkaistu kaikkina aikoina determinisesti ja se on jaettuna osiin, jotka vastaavat kutakin kandidaattia siihen vaikuttamassa. Jos mittaus alkaa annetusta alkutilasta (ollen sitä ennen nolla kaikkina aikoina), ei vielä voida sanoa, että tämä on vuorovaikutus, joka määrää lopputuloksen varsinaisen tuloksen. On myös vaihdettava alkutilaa ja mieluiten seurattava aikaevoluutiota menneisyyteen ja tiedettävä onko alkutila määräytynyt merkittävästi esim. pienten h:iden avulla. Ei ole näissä kohdissa mainittu sitä, voisiko moni romahdus kuitenkin alkaa iso-h objekin populaation välisistä vuorovaikutuksista tapahtuen aina silloin tällöin ja ollen valmiina. Jos näin on niin silloin varsinkin nämä alkutilat eivät koskaan ole olleet irrallisia, ja mikä populaatiossa mittaa kenet perustuu pitkään evoluutioon muiden h:iden kanssa.

    Yhden muodostetun systeemin käyttäytyminen siten, että siinä on kaikkien noiden kohtien jälkeen syntynyt herkästi mittausmenetelmiin reagoivia muuttujia kaikkina sen systeemin aikoina, ja muutkin objektit lähestyvät pienen hetken päästä dekoherenssia, on täysin anomaliatasolla huomioitu käytös, eikä muuta sitä, että alla toimiva mekanismi ei tunnusta mitään erikoisia objekteja tai vuorovaikutuksia.

    >>>>>>>>>> TGD:ssä tajunnalla olisi kaksi komponenttia

    Tykkää

  4. Teksti on aina katkennut kohdasta
    ”…aikakehittyvän Hamiltonin funktion osa on kolmannen kertaluvun Dysonin-kehitelmä, joka suppenee, kun B …”
    Tämä on puuttuva osa ilman le-symbolia:

    Häviävää kenttää ja erillisiä vuorovaikutuksia varten kirjoitin tiheysmatriisin kahdelle spin1/2-hiukkaselle, joista vain toiseen vaikuttaa x-suunnassa ajastariippuva magnettikenttä be^Bt ja spinien välillä on silti yhtäaikaa Helmholtz-kytkentä vakiolla a. Diracin aikakehityskuvassa pidän kokonaisaikaoperaattoria näiden osien matriisieksponenttien tulona ja aikakehittyvän Hamiltonin funktion osa on kolmannen kertaluvun Dysonin-kehitelmä, joka suppenee, kun B pienempikuin 0 ja kenttä poistuu. Nämä valinnat voivat määrittää, miten kaksi eri aikakehitelmä oikeasti suhtautuvat toisiinsa, eikä tätä voi pitää kuin häiriö-approksimaationa. Lopputuloksessa a:n suurudella ei esiinny taajuuden lisäksi muuta vaikutusta. Jos alkutilat ovat tilojen -+ ja ++ superpositio, missä edellisen matriisielementti on alussa q ja jälkimmäisen 1-q, nämä samat matriisielementit kehittyvät seuraavanlaisesti

    y=-((b^{6}*q*e^{6*t*B}-b^{6}*e^{6*t*B}-6*b^{6}*q*e^{5*t*B}+6*b^{6}*e^{5*t*B}-21*b^{4}*q*B^{2}*e^{4*t*B}+12*b^{4}*B^{2}*e^{4*t*B}+15*b^{6}*q*e^{4*t*B}-15*b^{6}*e^{4*t*B}+84*b^{4}*q*B^{2}*e^{3*t*B}-48*b^{4}*B^{2}*e^{3*t*B}-20*b^{6}*q*e^{3*t*B}+20*b^{6}*e^{3*t*B}+72*b^{2}*q*B^{4}*e^{2*t*B}-36*b^{2}*B^{4}*e^{2*t*B}-126*b^{4}*q*B^{2}*e^{2*t*B}+72*b^{4}*B^{2}*e^{2*t*B}+15*b^{6}*q*e^{2*t*B}-15*b^{6}*e^{2*t*B}-144*b^{2}*q*B^{4}*e^{t*B}+72*b^{2}*B^{4}*e^{t*B}+84*b^{4}*q*B^{2}*e^{t*B}-48*b^{4}*B^{2}*e^{t*B}-6*b^{6}*q*e^{t*B}+6*b^{6}*e^{t*B}-36*q*B^{6}+72*b^{2}*q*B^{4}-36*b^{2}*B^{4}-21*b^{4}*q*B^{2}+12*b^{4}*B^{2}+b^{6}*q-b^{6})*(\cos4*a*t+1))/(2*(b^{6}*e^{6*t*B}-6*b^{6}*e^{5*t*B}-3*b^{4}*B^{2}*e^{4*t*B}+15*b^{6}*e^{4*t*B}+12*b^{4}*B^{2}*e^{3*t*B}-20*b^{6}*e^{3*t*B}-18*b^{4}*B^{2}*e^{2*t*B}+15*b^{6}*e^{2*t*B}+12*b^{4}*B^{2}*e^{t*B}-6*b^{6}*e^{t*B}+36*B^{6}-3*b^{4}*B^{2}+b^{6}))

    y=(b^{6}*q*e^{6*t*B}-6*b^{6}*q*e^{5*t*B}-21*b^{4}*q*B^{2}*e^{4*t*B}+9*b^{4}*B^{2}*e^{4*t*B}+15*b^{6}*q*e^{4*t*B}+84*b^{4}*q*B^{2}*e^{3*t*B}-36*b^{4}*B^{2}*e^{3*t*B}-20*b^{6}*q*e^{3*t*B}+72*b^{2}*q*B^{4}*e^{2*t*B}-36*b^{2}*B^{4}*e^{2*t*B}-126*b^{4}*q*B^{2}*e^{2*t*B}+54*b^{4}*B^{2}*e^{2*t*B}+15*b^{6}*q*e^{2*t*B}-144*b^{2}*q*B^{4}*e^{t*B}+72*b^{2}*B^{4}*e^{t*B}+84*b^{4}*q*B^{2}*e^{t*B}-36*b^{4}*B^{2}*e^{t*B}-6*b^{6}*q*e^{t*B}-36*q*B^{6}+36*B^{6}+72*b^{2}*q*B^{4}-36*b^{2}*B^{4}-21*b^{4}*q*B^{2}+9*b^{4}*B^{2}+b^{6}*q)/(b^{6}*e^{6*t*B}-6*b^{6}*e^{5*t*B}-3*b^{4}*B^{2}*e^{4*t*B}+15*b^{6}*e^{4*t*B}+12*b^{4}*B^{2}*e^{3*t*B}-20*b^{6}*e^{3*t*B}-18*b^{4}*B^{2}*e^{2*t*B}+15*b^{6}*e^{2*t*B}+12*b^{4}*B^{2}*e^{t*B}-6*b^{6}*e^{t*B}+36*B^{6}-3*b^{4}*B^{2}+b^{6})
    https://www.desmos.com/calculator
    -formaatissa.

    Tilan +- todennäköisyys on sama kuin q(t+v/2), missä v on a:sta riippuva aallonpituus. — lähestyy nollaa kaikkina hetkinä. B on sekä kytkentä, suuruus että keston määrä, mutta b parametrisoi pelkästään ensimmäistä, jos halutaan. Suuret b-arvot johtavat jyrkempiin kehityksiin kohti lopputilaa, joka on saavutettu viimeistään, kun t=10/bB, mutta sisältää eri välejä sadasosien välissä, missä tilat tai ++ vaihtelee lineaarisesti nollasta yhteen ja saattaa vielä muodostaa uuden aallonhuipun b-resonaatiossa. Ero systeemien a=0 ja a!=0 välillä on se, että edelliset päätyvät aina stabiileiksi, mutta muutoin -+ ja +- tilojen on lähestyttävä nollaa taikka niiden resonaatio jää jäljelle käyden aina a=0 -tapauksesta nollaan.

    Huom. että yhdessä äänessä sanotaan ikäänkuin energian kasvavan, samalla kun h:t kasvaa, mutta ennenkuin tulin kertomaan aallonpituuden merkityksestä tällä on haluttu saada aikaiseksi varmaan kerralla kaikkien energian saamien arvojen kasvu (potentiaali ero) stabiileissa tiloissa ja perustuen vain kytkentävakioihin. Näin ei oikeasti kävisi millään energian syömisellä, vaan jos kvanttiobjekti pakotetaan ottamaan energiaa vastaan vuorovaikutuksessa, se joko luhistuu ylemmälle energian aaltofunktiolle tai sen ylempien tilojen todennäköisyys kasvaa, jos enempää tiloja ei ole niin energia menee ohi tai alkaa tapahtua muuta hiukkasfysiikkaa. Jos koko vieraan ulottuvuuden energiatiheys muuttuu, ovat jotkin hiukkaset ehkä parempia kuin toiset ja kytkeytyvät eri tavalla.

    Lopulta jos vuorovaikutus näkyy jossain ensimmäisenä niin kvanttimekaniikassa katseltaisiin silloin alkioita tiheysmatriisissa, joka on ratkaistu kaikkina aikoina determinisesti ja se on jaettuna osiin, jotka vastaavat kutakin kandidaattia siihen vaikuttamassa. Jos mittaus alkaa annetusta alkutilasta (ollen sitä ennen nolla kaikkina aikoina), ei vielä voida sanoa, että tämä on vuorovaikutus, joka määrää lopputuloksen varsinaisen tuloksen. On myös vaihdettava alkutilaa ja mieluiten seurattava aikaevoluutiota menneisyyteen ja tiedettävä onko alkutila määräytynyt merkittävästi esim. pienten h:iden avulla. Ei ole näissä kohdissa mainittu sitä, voisiko moni romahdus kuitenkin alkaa iso-h objekin populaation välisistä vuorovaikutuksista tapahtuen aina silloin tällöin ja ollen valmiina. Jos näin on niin silloin varsinkin nämä alkutilat eivät koskaan ole olleet irrallisia, ja mikä populaatiossa mittaa kenet perustuu pitkään evoluutioon muiden h:iden kanssa.

    Yhden muodostetun systeemin käyttäytyminen siten, että siinä on kaikkien noiden kohtien jälkeen syntynyt herkästi mittausmenetelmiin reagoivia muuttujia kaikkina sen systeemin aikoina, ja muutkin objektit lähestyvät pienen hetken päästä dekoherenssia, on täysin anomaliatasolla huomioitu käytös, eikä muuta sitä, että alla toimiva mekanismi ei tunnusta mitään erikoisia objekteja tai vuorovaikutuksia.

    >>>>>>>>>> TGD:ssä tajunnalla olisi kaksi komponenttia….

    Tykkää

  5. >>>>>>>>>> Systeemin jako kahdeksi erilliseksi kvanttikietoutumattomaksi ali-systeemiksi on tarkalleen se mitä tapahtuu tilafunktion reduktiossa joka taas on tietoisen kokemuksen perus-elementti.
    Mikä tahansa Hilbert avaruus jonka dimensio ei ole alkuluku voidaan jakaa tensorituloksi. Alku-lukudimensioiset Hilbert avaruudet ovat itse asiassa erityisen mielenkiintoisia, koska ne ovat tavallaan jakamattomia alkeis-systeemejä.>>>>>>>>>>

    (Sanotko nyt ettei ole ylä- ja alasysteemeitä yhtä aikaa eikä mitään muuta suhdetta kuin esim. systeemi joka seuraa edellisestä systeemistä? Miksi silloin voisi aloittaa yläsysteemistä? Teen pitkiä osoituksia lähinnä siitä, että alin systeemi eli yksi objekti on aina, ja on muiden objektien kanssa täysin suvereeni.)

    ”Tarkalleen” ei voi olla yksi-yhteen-vastaavuus, koska myös yhden objektin yhden tilan tilafunktio romahtaa, eikä mahdollisia funktioita ole sen jälkeen useampia. Tällainen jakaminen ei myöskään ole yhtään oleellista Hilbertin avaruuksille, jos ne on muodostettu yhdelle muuttujalle kerrallaan ja muuttujien olemassaoloa yhdessä vastaa tensoritulot annetuista avaruuksista. Myöskään jossain imaginääriluvuista reaalisiin mentäessä ei niissäkään jaeta välttämättä alussa annettua avaruutta vaan määritellään kokonaan uusi (eikä saavuteta uutta suurettakaan muutenkuin saman suureen kantavektorien kasvamisena itse vektorien jne. vektoriksi). En pidä tilafunktioiden romahtamista ja kietoutumista hyvänä perusteena systeemin jakautuvuudelle ja kerron välillä lisää tilojen yhdistämisestä. Romahdus ja kietoutuminen kuvaavat vain tietyn (melkein sovitun) määritelmän mukaisia tiloja. Nämä yksittäiset tilat eivät voi rikkoa periaatteita siitä, että minusta huom. kietoutuneessa tilassa on kaksi systeemiä, ja että minusta romahtaneessa tilafunktiossa on kaksi systeemiä samoin kuin ennen romahtamista, koska kaikki objektit ovat koko ajan osa yhtä mekanismia ja formalismia, joka merkitsee enemmän niiden järjestelemiseen kuin yksittäiset tapaukset.

    Kietoutumisen määritelmä annetaan yleensä siten, että joss yhdistettyä tilaa ei voida esittää kahden tilan tensoritulona, se on kietoutunut (joskus tämä on kirjoitettu vain ket-tiloille, mutta pätee myös tiloille, jotka on esitettävä tilojentiheydellä). Tila, joka on (vasen tila) o (oikea tila), tarkoittaisi, että kaikki vasemmalla puolella olevat arvot voivat esiintyä aina kaikkien oikean puolen arvojen kanssa. Vasemalla ovat voimassa vain ne arvot, joita vastaavat kantavektorit esim. mudostavat vasemman tilan summana sen ket-tiloista. Tilat tuottavat tuloksia täysin toistensa tuloksista riipumatta. Tai tarkemmin sanottuna: tilastollisesti ne eivät korreloi, vaikka ne olisivat jostain arvatun keskeisriippuvuuden mudostamasta hyvästä syystä kirjoitettu yhdistetyksi, mutta näissä tapauksissa sellainen ei olisi tuottanut korrelaatioita tarkastellulle hetkelle. Hilbertin avaruuksien välinen tensoritulo kuvaa kaikkia määriteltävissä olevia suureiden arvoja kerralla, ja jos vasen ja oikea tila ovat kaikkien kantavektoriensa summia, niiden yhdistetty tila jossain mielessä on kaikkia arvoja täynnä yksillä todennäköisyyksillä, mutta ei silti viritä koko vektoriavaruutta, kun jokainen puhdas tila on yhdistetyn Hilbertin avaruuden puolisuora. Hilbertin avaruuksien tensoritulo sisältää siis tätäkin enemmän objekteja sekä puolisuorina että muissa muodoissa. Hilbertin avaruutta voi tällöin pitää yhden objektin systeeminä, joka pysyy samana kaikkina hetkinä eikä ole orja kenellekään. Hilbertin avaruuksissa ei silti missään määritellä vuorovaikutuksia, eikä objektia voi pitää minusta minään ilman niitä. Millaiset vuorovaikutukset ovat, on myös aina pysyvä ominaisuus. Tämä määrittely johtaa vaikka mihin yhden objektin kannalta, kun päädyn lopulta Hamiltonin operaattoriin ja kietoutumiseen. Kirjoitan seuraavaksi tiloista ja operaattoreista lisää, jos ei niidenkään kanssa pitäisi nähdä mitään ylä- ja alakäsitteitä.

    Yhdistettyjen tilojen yksi erikoistapaus on siis tila, joka on yhdistetyssä Hilbertin avaruudessa (pysty-) vektori. Näitä tiloja muodostuu vain laskemalla yhteen ket-merkinnän omaavia kantavektoreita valitussa kannassa. Yhdistetyn systeemin ket-vektorit ovat teoriassa osittaistilojen ketien tensoritulo(-n alkioiden vektori), mutta on mahdotonta esittää yleistä yhdistetyn systeemin lineaarikombinaatiota tai superpositiota sen vektoreista vain kahden osittaisen vektorin tulona. Samoin kuin ei voida virittää R^4 avaruutta valitsemalla sen aliavaruuksien, esim. kahden R^2 tason vektoreista kahta ja tekemällä niistä yksi R^4:n vektori. Se että yhdistetty tila on puhdas ja vektori ei riitä määrittämään, että tila on kietoutunut, koska yhden kantavektorin esittämät tapaukset, joissa ei ole kuin yksi vaihtoehto suureilla, ei sisällä yhtään korrelaatioita, eikä sekään tila missä kaikki kantavektorit samalla todennäköisyydellä. Puhtaan superposition pitäminen kaikkein kietoutunempana tilana muuhun verrattuna pistää jotkut myös määrittelemään erikseen klassisen ja kvanttimaisen korrelaation, mitkä molemmat tarkoittavat vain sitä, että kaikki suureet eivät saa kaikkien mahdollisia arvojen välisiä kombinaatiota, vaan että näihin mahdollisuuksiin ilmestyy aukkoja.

    Edellä mainittu ”Kaikki koherenssi kahden objektin välillä perustuu joissain selityksissä tensoritulossa muodostuneisiin epädiagonaali-alkioihin, mutta tämä on hyvin herkkää kannanvaihdolle, eikä oikein tuota mitään selvää, sen on varmaan enemmän esiinnyttävä yhdellä objektilla sen osittaistilan tasaisena diagonaalina” on oikeammin sanottuna niin, että epädiagonaali-alkiot erottavat toisistaan klassisia ja kvanttikorrelaatioita jopa perustavasti, mutta tarkoitan lopulta näiden ja kietoutumisten suuruuksien havainnollistamista, mikä on joskus ongelmallista mielivaltaiselle tilalle. Tietäisin itse, miten korreloitunut tila on, jos operoisin yhteen tilaan projektiomittauksilla, ja laskisin miten paljon toisen tilan todennäköisyysjakaumasta aidosti häviää tai miten paljon eri tulosten välinen suhde muuttuu, mutta operoinnin takia suurin osa yhdistetyn tilan epädiagonaalialkioista ei vaikuta tulokseen vaan monet korrelaatiot käyttäytyvät huom. samoin.

    Jos tutkitaan yhdistettyä tilaa superpositiona, esim. sen mahdollinen romahtaminen suoritetaan operoimalla toiseen tiloista käyttämällä operaattoria, joka on tensoritulo yksittäisen objektin operaattorista ja identitettioperaattorista, joka ei vaikuta jälkimmäiseen tilaan. Kaikki yhden objektin fysikaaliset suureet ovat tällaisia operaattoreita, ja suure voidaan määritellä yhdistetylle systeemille, ja se on olemassa yhdisteneisyyden aikana omaten yhtä hyvän odotusarvon määritelmän kuin itsenäisessä kvanttitilassa, jossa arvoilla on todennäköisyysluonne.Ilman operaattorisuuretta ei ole nykyaikana mielekästä määritellä tiloja eikä siis määrätä systeemien aliylitystä ilman niitäkin. Operaattorit eivät ole aina kaikki yhtä stabiileja kuin yllä mainittu Hilbertin avaruus, mutta suureen määrittävä operaattori on hyvin erityinen. Seuraavassa jaetaan operaattoreita samalla tavalla kuin tiloja.

    Operaattorit (vOp) o (oOp) eivät koskaan koe (minkään tilan tapauksessa), että ei olisi olemassa kahta erillistä systeemiä, koska aluksi ne erottavat toisistaan superpositiossa olevat yhdistetyn tilan eri kantavektorit ja operoivat kuhunkin näistä (distributiivisuus). Kantavektorit ovat muotoa (vasen) o (oikea) ja sitten operoidaan tiloihin, joiden tiedetään olevan itsenäisen systeemin muotoisia. Schrödingerin kuvassa tai muuten pitämällä operaattoreita vakioina ei tilan muutos koskaan aiheuttaisi sitä, että niiden alisysteemiys Operaattorit voivat kuitenkin olla eri muotoa kuin (vOp) o (oOp) ja niitä muodosteaan samoin useiden operaatoreiden summista. Distributiivisuus pätee myös operaattoreidenkin puolelta, jolloin on lopulta joukko osittaisoperaattoreita, jotka vaikuttavat oman Hilbertin avaruutensa vektoreihin. Kaikilla mahdollisilla operaattoreilla ei ole myöskään sellaista hajotelmaa, joissa jokainen kantamatriisi olisi osittaisen tilan operaattori ja identiteetti tensoritulossa.

    Yhdistetty systeemi voi generoida joukon ylimääräisiä suureita riippuen millaisia operaattoreita ja tiloja Hilbertin avaruudessa on (yhdistetyt suureet ovat olemassa riippumatta kietoutumisesta). Yksinkertaiset näistä ovat summia erillisistä suureista (kokonaisenergia jne.), jotka eivät vaikuta osatiloihin tai käyttäydy niiden erillään esiintyvästä käyttäytymisestä poiketen. Lopuille suureille pätee jonkinlainen kaava, missä niillä on oltava ominaisarvot ja -vektorit, jotka lopulta perustuvat osittaistilojen kantavektoreihin. Uusien epäkommutoitvien suureiden syntyminen voisi merkitä uutta systeemiä, mutta tätä systeemiä ei ole olemassa erikseen koko maailman systeemissä, vaan sen suureet esiintyisivät jonakin erilaisena operaattorijoukkona.

    Yhtenä tuloksena voidaan esittää, että jos tilojen superpositiosta tehdään periaate, ettei alisysteemiä ole silloin olemassa, kun sen kantoja on eri tuloissa ja tuloja laskettu yhteen, tästä voitaisiin tehdä yleistys operaattoreille, joiden alisysteemiä ei ole olemassa, kun esim. yhteinen Hamiltonin operaattori ei ole muotoa (vH) o (oH). Nyt käy kuitenkin niin, että maailma muodostuu yhdestä Hamiltonin funktiosta, jolla on useimmille klassisille objekteille hajotelma (vH) o (I) +…, mutta jo kahden tällaisen operaattorin summa pienemmälle maailmalle ei ole esitettävissä yhtenä tensoritulona ja jouduttaisiin sanomaan ettei maailmassa ikinä voi olla systeemejä kuin yksi kappale. (vH… voi olla mikä tahansa suuretta vastaava operaattori, hermiittinen mutta täysin yleisessä muodossa, ja tutkitaan näiden suureiden summaa maailmassa. Silti mistä tahansa vakioenergiaoperaattoreista tehty unitaarioperaattori on aina tensoritulo).

    Osittainen jälki voidaan ottaa yhdistetystä tilasta, ja se sisältää ikäänkuin yhden tiheysmatriisin muodossa koko tilastollisen jakauman, joka yhdellä objektilla on sekä jotain, mistä joskus katsotaan onko osittaistila puhdas tai sekoittunut, jos matriisia vastaa vektorimuoto. Saatavat tilat eivät voi sisältää korrelaatioita ja eri korrelaatiot eivät tuota uniikkia os. jälkeä, eikä kahdesta jäljestä voi muodostaa käänteisessä mielessä uudestaan tensoritulolla tilaa, joka olisi sama kuin alussa.

    Vuorovaikutukset Hamiltonin operaattorin muodossa ovat tensoritulojen operaattorien summa. Vain jotkut H:t ja sellaiset, jotka eivät ole muotoa (vH) o (I)+(I) o (oH), voivat aikakehityksessä muuttaa kietoutumattoman tilan kietoutuneeksi tai päinvastoin. Yleensä kietoutumisuuden välillä liikutaan formalismeissa vain operaattoreilla, jotka ovat projektioita tai yksinkertaisia matemaattisia luomuksia. Joistakin näistä kuitenkin näkee sen, että kietoutumisen syntyminen nollakorrelaatio-alulle on mahdollista vain jos siihen operoidaan summalla operaattoreita, joista jokin on myös molempien tilojen operaattori (vOp) o (oOp) (joista mikään ei yksin riitä). Aikakehitysoperaattorit voivat olla tällaisia teoriassa, mutta kirjoittamalla unitaarioperaattorin yleisessä muodossa voidaan todeta ettei ole ratkaisuja, jotka tekevät muunnoksen. Millekään äärimmäisille kietoutumisen muutoksille ei siten tunneta yksinkertaisia vuorovaikutuksia ja suurin osa tutkimuksista liittyy makrotason ympäristöön, epähermiittisiin H:ihin ja kääntymättömään aikakehitykseen ja näistäkin monelle riittää kehitys helpompien tilojen välillä. Tämän kaiken pitäisi johtaa lähinnä siihen, että mikään yhdistetty systeemi ei voi itsestään kehittyä mihin tahansa kietoutuneeseen tilaan, eikä sitä voi pitää silloin suvereenina suvereeniuden saavuttamisessa, vaan systeemi tarvitsee jonkin ulkopuolisen kentän tai objektin ollakseen koskaan edes sellainen, missä sanot ettei sillä ole alitiloja. Hyvällä tuurilla jokin alimmaisin tila tai yksi objekti, joka on identtinen kahden muun alitilan kanssa, tai on myös paljon pienemmän planckin vakion objekti, voi kietouttaa nämä edeltävästi ennen kutakin tietoisuudeksi haluttua prosessia.

    >>>>>>>>>>> Näyttäisi siltä, että kirjoittaja uskoo aaltofunktion dekoherenssin käsitteeseen joka on yksi yritys tulkitsemalla pelastaa kvanttimittausteoria. Näitä pelastusyrityksiä on valtaisa määrä ja näyttää vahvasti siltä että potilas on kuollut ajat sitten.

    Dekoherenssimallissa ei millään tavalla spesifioida miten dekoherenssi tapahtuu. TGD:ssä lähdetään nimenomaan siitä että kvanttimittaukset voivat tapahtua mille tahansa alisysteemille jakaa sen kvanttikietoutumattomaksi tensorituloksi. Universaali observaabeli on tiheysmatriisi.>>>>>>>>>>>

    Dekoherenssilla olen tarkoittanut tilojen romahtamista deterministisesti ja paljon tai vähän jossain ympäristössä, mikä ei ole sanan ensimmäinen määritelmä. Pelkän (de)koherenssin evoluutio on täydelle dekoherenssille sellainen, että puhtaasta tilasta tulee diagonaalinen tiheysmatriisi ja kvanntikorrelaatioista klassisia korrelaatioita, jotka ideaalisesti eivät myöskään painotu mitenkään uudella tavalla ja mahdolliset mittaustulokset jäävät kokonaan muuttumatta. Jokainen yksittäinen malli jollekin evoluutiolle kuvaa tämän dekoherenssin kyllä täysin teoriassa ja hyvin, koska systeemit voivat olla yksinkertaista muotoa, ja moni malli varmasti on myös joissain kokeissa vastaavasti olemassa (mutta täytyy todistaa havaitsemalla koherenssi). Moni kvanttiobjektin ympäristöön kytkevä malli, joka muuttaa myös mittaustodennäköisyyksiä deterministisesti kohti romahdusta perustuu samoihin ideoihin kuin niitä edeltäneet mallit. Tai jos olet oikeassa, sitten on todennäköisesti käynyt niin, että vasta kun dekoherenssille piti saada malli, joka perustuu oikeisiin vuorovaikutuksiin, niin se ei ollut enää pelkkää dekoherenssia. Ainoa ideologinen seuraus dekoherenssisysteemeistä tapahtui silloin kun ne varmaan ensin tulivat ja käsitteenä sen nimi on ”many worlds theory”.

    Kun ensimmäisessä komentissa lukee sekoitettu tai superpositio-tila, ei ole myöskään mahdollista oikeasti sulkea toista tapausta pois, vaan olisi pitänyt kirjoittaa yleinen tila yhdistetyssä systeemissä. Silti esimerkin tila on varmasti ei-superpositio kaikilla arvoilla q. Ja siinä olevalla spin-spin -kytkennällä (a) ei ole olemassa henkilön nimeä, ja jotkut Helmholtzista spintilojen yhteydessä puhuvat paperit tarkoittavat sillä jotain ulkoista kenttää.

    >>>>>>>>>>> TGD:n inspiroima tietoisuuden teoria palauttaa vapaan tahdon tilafunktion reduktioon ja samalla ratkaisee kvanttimittausteorian perusongelman.
    Esimerkiksi Minev et al:n tekemät kokeet atomifysiikan tasolla antavat suoraa tukea varsi dramaattisille ennustuksille.
    Samoin Libetin havainnot.>>>>>>>>>>>

    Minevin ”To catch and reverse a quantum jump mid-flight” (2019, arxiv: 1803.00545)

    on kuin mikä tahansa mainituista todennäköisyyksiä muuttavasta systeemeistä. Yhtä Supplementaryssä olevista malleista kutsutaan koherentiksi, koska sen aikakehitys voidaan antaa ket-vektoreille ja suurten kenttien/kytkentöjen approksimaatiossa näiden kertoimet toteuttavat täyden mittaustapahtuman (mutta suljetussa muodossa ei voitu osoittaa, että aina). Tämä ei ole kaikkein realistisin vaihtoehto vaan artikkelia tehdään myös applikaatioille, joissa halutaan vähentää koherenssin häviämistä ympäristöön systeemeissä, joissa näin käy, ja aina ennen mittausta, koska sitä pidetään virheenä. Kun seuraava malli ei tuota täyttä mittausta, tätä ei kannata pitää sen ”inkoherenttisuuden” syynä. Päätekstissä esim. s.5 koherentiksi kuvataan mahdollista löytynyttä mittausteoriaa, jolla on ensin oltava evoluutio, minkä yhteydessä kööpenhaminan tulkintaa ei ikäänkuin pidetä koherenttina, mutta kööpenhamina tai von Neumannkin päätyy täyteen mittaukseen.

    >>>>>>>>>>> Vapaan tahdon olemassaoloa ei varmaan tullakaan koskaan todistamaan matemaattisesti. Kuten ei mitään muutakaan fysiikassakaan oletettua. >>>>>>>>>>>

    Vapaata tahtoa ei ole oletettu missään.

    >>>>>>>>>>> [MP] Tässä on taas massiivinen väärinkäsitys kyseessä. Kirjoittaja lähtee omista oletuksistaan joiden mukaan klassinen fysiikka on kvanttifysiikan raja pitkissä pituus-skaaloissa. TGD:ssä (nolla-energia-ontologiassa) tilanne on täysin toinen. Kvanttitilat ovat superpositioita klassisista deterministisistä aikakehityksistä: 3-D aika=vakio snapshot kvanttitilana on korvattu superpositiona 4-D avaruusaikalehdistä kausaalitimantin sisällä..>>>>>>>>>>>

    Jos tietoisuuden pienimmän h:n ja kehon osa on kvanttitiloissa, niin sitten on vain kvanttitilat, jotka mittaavat kvanttitiloja. Mutta näiden mittaamaa informaatiota on vaikeampi säilyttää ja muodostaa muistia mittauksista. Kvanttimuiston uudelleen tiedostaminen kvanttitietoisuudella jne.

    En ole pitänyt pituuksia minään, vaan sitä että joskus kvanttiobjektille tulee enemmän kvanttiobjekteja vastaan. Mitään muuta selitystä ei ole sille, että omankin selityksesi pitää sitten selittää miksei kaikessa aineessa ole olemassa montaa determinististä aikakehitystä kerralla? Vosiko niiden määränkin selittää deterministisesti? Ei silti kannata ajatella tämän antamaa tulosta (ykköstä) mekaniikkojen rajana.

    Kuinka monta dimensiota on yhdessä lehdessä, maksimidimensioisen CD:n tapauksessa? Koska jos niitä on kolme avaruudellista dimensiota, niin superpositiossa on snapshotteja eikä aikakehityksiä ainakaan tämän avaruuden ajalla. Yhdistetty systeemi on varmaan snapshot eikä 4D-tilojen tensoritulo, vaikka jälkimmäistä ei voi estää?

    >>>>>>>>>>> [MP] Kompleksiluvut ovat rationaalinen täydentymä jossa algebralliset luvut vielä täydennetään jatkumoksi. Mitä tulee havaitsemaamme fysiikkaan, emme voi tietenkään olettaa että se vastaisi tätä ideaalista rajaa. On kyse myöskin havaitsijan kuvaamisesta, ei minään ideaalisena muukalaisena vaan osana systeemiä. Kognitiiviset esityksen ja laajennusten hierarkia evoluutiohierarkiana tulevat tällöin mukaan.>>>>>>>>>>>

    Kompleksiluvuista ja havaitusta fysiikasta voi sanoa sen juuri, että todellinen maailma ei niitä välttämättä kaipaa. Se käyttää vain osaa ns. algebraryhmistä ja teoriat, joissa on ylimääräisiä ryhmiä, joutuvat hankkiutumaan niistä eroon kussakin tapauksessa. Nimenomaan se että kompleksiluvuilla on suljettu algebra, kun suljettavuus ei ole vaadittu ominaisuus (tätä ei tiedetä, jos se kuitenkin on pakollista kaikenteoriateorioissa, jotka on jo kiinnitetty niin fyysisiksi kuin voi), voi olla epäfyysistä siksi ettei maailmassa ole negatiivisia lukuja ja kaikki reaalifysiikka käyttää reaalilukuja myös jonkinlaisena valittuna mittana.

    Ajatusharppauksesi ei voi tarkoittaa, että luvuista päädyttäisiin mittaamisprosessiin, koska osana systeemiä oleminen on fyysistä todellisuutta ja jää jäljelle vaikka esim. ideaalinen kompleksilku-teoria sen esittäisi. Saman esityksen muuttuminen eri luvuiksi tai ryhmiksi ei vaikuta suoraan siihen, onko esityksessä mukana näitä systeemin osia.

    >>>>>>>>>>> Tällä tavalla klassinen teoria- avaruusaikapintojen geometrodynamiikka – ja myös Diracin yhtälön yleistys – ovat eksakti osa teoriaa.Klassinen teoria ei ole todellakaan approksimaatio. Kvanttihypyt tapahtuvat näiden superpositioiden välillä. Klassinen deterministinen kausaalisuus pätee eikä johda ristiriitaan kvanttihypyn ei-deterministisyyden kanssa. Tämä on koko teorian kuningasidea.>>>>>>>>>>>

    Minevin artikkeli ja muutkin yrittävät osoittaa kvanttihyppyjen olevan deterministisiä. Hänen teoriassaan on 3D Galilein avaruus. Deterministisessä kvanttihypyssä odotetaan aika dt >> 0, ja lineaarikombinaatiossa (vektorien tai tiheysmatriisien) ei ole jäljellä enää useita tiloja. Voitko nimittää mainitsemiasi hyppyjä tästedes 4D-hypyiksi, jos niistä nimittäin tulee joko dt=0 -rajalla tai mieluiten Galilein-approksimaatiossa samanlaisia deterministisiä 3D-hyppyjä?

    Jos TGD on geometrodynamiikkaaa, sen 3D-lehtiavaruudet olisivat metriikaltaan indusoituja 4D-avaruudesta.

    >>>>>>>>>>> [MP] Ei-relativistisen kvanttifysiikan kuvailu perustuu Hamiltonin käsitteeseen. Se on ok niin kauan kuin Newtonilainen kuva on hyvä approksimaatio ja varmastinen hyvä fenomeologinen työkalu yksinkertaisten systeemien kuvailuun. Siitä ei ole kuitenkaan apua TGDssä.

    Pistemäinen hiukkanen korvataan 3-pinnalla, jonka rata määrittelee avaruusaikapinnan alueen. Säiemallin 1-D säie korvautuu 3-D pinnalla. Vaati kuutisen vuotta vakuuttautua siitä, että kvanttikenttäteorioiden polkuintegraali ei todellakaan toimi. Lopputulos oli käsite ”klassisten maailmojen maailma” avaruusaikapintojen muodostamana ääretön-D Kaehler geometriana, joka pelkästä olemassaolostaan on uniikki jo luuppiavaruuksien tapauksessa, plus ”nolla-energia-ontologia” joka korvaa Newtonilaisen ontologian. Siinä yleinen koordinaatti-invarianssi pakottaa identifoimaan kvanttitilat superpositioina avaruusaikapinnoista jotka ovat analogisia Bohrin radoille.>>>>>>>>>>>

    3D-kehitykset, joiden avaruuksien on sanottu nauhoissa olleen ei-kaarevia, määrittävät kuitenkin yllä jokaisen TGD:n ratkaisun ja lopputilan, ja TGD lisää vain niiden probabiliteetit, ja osan systeemin dynamiikasta siinä mielessä, että superpositiotila muuttuu hypyn aikana? Jos 4D-avaruudessa on avaruudenkaltaisia lehtiä, ja niiden kaarevuus ei muutu lehtien välillä, kun aika muuttuu, niin avaruuden puolesta systeemissä on mahdollista olla millä tahansa hiukkasta ja timanttia ylemmällä tasolla säilyviä suureita ja Hamiltonin mekaniikassa on jopa paljon järkeä, jos suureille ei tapahtuisi mitään muutoksia 4D-superpositioissa (ehkä tämän pitää seurata TGD:n Galilei-approksimaatiosta), joka estää nimenomaan kvanttiobjekteja olemasta esim. energialtaan säilyviä. Et ole kertonut miten 4D-superpositio muuttuu tai tulkitaan muuten kuin pitämällä sitä epädeterministisenä, mikä yleensä tarkoittaa kaikkien invarianssien puuttumista objektin suureista ja yhtälöistä, koska yhtälöjä ei hypyn aikana ole. Koordinaatti-invarianssia ei voi pakottaa millekään paitsi havainnoille, joista ollaan erittäin varmoja. Onko tässä oltava varma joistain tavallisen kvanttimekaniikan arvon tai yhtälön invariansseista? Miten se on johtanut kokonaan omaan romahtamisdynamiikkaan? Jossain näin, että oli tehty koordinaattimuutoksista myös kvanttitiloja ja suureita, jolloin kokonaistila oli yksi ja sama superpositio näiden kanssa. Ja muunnoksilla oli Hilbertavaruus ja Hamiltonilainen kehitys. Tämä vastaa hyvin paljon mittakenttien lisäämistä hiukkastilaan.

    Kvanttigravitaatiot, paitsi säieteoria, ovat yleistetyn Hamiltonin mekaniikan teorioita, missä voi olla 3D Hamiltonin funkiton asemasta 4D-funktioita ja Hamiltonin-ehtoja. Idea siitä, että gravitaatiolle yleisen 4D-avaruuden sijaan riittää tarkastella (3+1)D avaruuksia liittyy siihen, että on kaksi kaareutumisen vapausastetta, joiden parametrisoitu evoluutio voi olla 3-ulotteisen metriikan (max. 3 vapaata muuttujaa jo tehdyn koordinaattien kiinnityksen jälkeen) evoluutio. Näihin kahteen asteeseen päätyminen ei ole välttämättä uniikkia, mutta täysin ekvivalenttia sen kanssa, että ne seuraavat yleistetystä Hamiltonin mekaniikasta, missä alkuehdot monissa tarkasteluissa näyttävät siltä kuin lehditys olisi tehty jo (ja kaikki tämä ajattelu kiertää kehää), mutta menetelmän pitäisi tarvita vain koordinaattimuuttujia, joista jotain pidetään ajan parametrina. Jos TGD ja kvanttigravitaatiot ovat molemmat totta, niin silloin TGD:ssäkin on enemmän H:n kaltaisia formalismeja löydettävissä (säieteoriaa ja H:hon perustuvia teorioita on osoitettu samoiksi ainakin ilman gravitaatiota).

    Jos 3-pintainen kvanttiobjekti lähestyy vapaan hiukkasen aaltofunktiota, sen aaltopaketilla ja keskiarvolla voi olla rata, mutta aaltoa ei voi määritellä kuin äärettömässä alueessa. Vapaan hiukkasem aaltofunktio ei ole vakio kuten Bohrin rata, eikä mitään t:stä riippuvaa pintaa voi enää pitää samanlaisena kuin avaruudenkaltaiset lehdet. Rataa joka 4D:hen tulee voidaan aina esittää jokaista aikaa vastaavina avaruuslehtinä, joista jokaisessa on objekti, mutta tällöin jokaisen lehden määritelmä on se, minkä se saa siivuteltavana olevasta 4D-avaruudesta. Muutenkaan tältä ei pitäisi pystyä välttymään, koska jos yrität silti tehdä makroavaruuteen toisenlaisia aliavaruuslehtiä jonkin objektin radasta (tangentin suuntaan parametrisoiden tms.) on tämä rata aina aluksi määritelty kyseisessä makroavaruudessa, eikä sen aliavaruudet ole erillisiä määritelmiä jossa ne ovat esim. klassisia. Muutkin käyttävät lehtiä joskus kuin niitä voisi käyttää lokaalisti eli vaikka 4D-avaruuden kaarevutta ei kokonaan tunneta joka pisteessä, mallissa on tällöin samoja heikkouksia kuin tavallisessa kvanttimekaniikassa, joka ottaa vain vähän ympäristöä huomioon.

    >>>>>>>>>>> [MP] Eri avaruusaikapintojen, ei eri aikojen. Nollaenergaontologiassa kvanttitila ei ole aika=vakio snapshot vaan superpositio klassisista deterministisistä aikakehityksistä, preferred extremals, jotka ovat analogisia Bohrin radoille.
    Tämä seuraa yleisestä koordinaatti-invarianssista: annettuun kolmipintaan liittyy uniikki nelipinta, jolla yleiset koordinaattimuunnokset 4-D mielessä vaikuttavat. Se implikoi taas holografian: datat 3-pinnalla kiinnittävät 4-pinnan.
    Itse asiassa holografia osoittautuu vahvemmaksi. 2-D data kiinnittävät 4-pinnan.>>>>>>>>>>>

    Aloit puhua pinnoista, koska objektit ovat hiukkasen sijasta pintoja, mutta tässä on myös 4-pinta, joka on lopullisen kvanttiobjektin esitys eikä lehti, jossa on suljettu pinta? Silloin tätä ns. 4D-superposition kiinnittämistä 3D tiloilla kannattaa verrata siihen etteivät Hilbertin avaruuden superpositiotkaan ole omia ylempiä systeemejään. Jos tässä taas ovat kyseessä koordinaattiavaruudet tai monistot ja alimonistot, niin niille ei ole menetelmää luoda suuremman dimension metristä tensoria pelkistä indusoiduista tensoreista, vaikka nämä kaikki tunnettaisiin esim. lehditetylle avaruudelle. Lehtien suunnan parametrin, josta tulee koordinaatti, metriikka täytyy antaa erikseen. Silti tietysti lehtien ja pienempien avaruuksien muodostus on vain matemaattista, ja jos gravitaatioteoriassa sanotaan jotain, mistä suuren avaruuden kaarevuus tulee, niin on samantekevää, onko sen lähteen muualla esitetty olevan lehdillä.

    >>>>>>>>>>> [MP ]M^8-H dualisuus merkitsee sitä että avaruusaika voidaan tulkinta pintana M^8:ssa tai H:ssa. M^8 kuvassa avaruusaika määräytyy polynomista rationaalikertoimin ja tämä polynomi määrää algebrallisen laajennuksen ja avaruusaikapinnan tason evoluutiohierarkiassa.
    Algebralliset laajennusten parametrit kuten dimensio n=h_eff/h_0 (h= 6h_0 seuraa empiirisesta argumentista) ja ramified primes ovat tärkeitä ja kytkentävakiot ja niiden diskreetti evoluutio laajennuksen funktiona määräytyisi näistä paramereista. On varmasti muitakin laajennusta kuvaavia lukuteoreettisia parametreja. Sironta-amplitudien lausekkeet yksinkertaistuvat, koska avaaruusaika diskretoituu uniikeiksi kognitiiviseksi esitykseksi. Tässä tulee mukaan lukuteoria, joka on tällä hetkellä voimakkaasti kehittymässä.>>>>>>>>>>>

    Miksi avaruusajan reaaliosien pitäisi laajentua algebrallisest? Jos siitä on hyötyä, niin miksei niitä laajenneta aina kompleksisiksi, missä laajennus päättyy? Monisto C^4 vastaa R^8 monistoa ja minkä tahansa pinnan funktiot saadaan C^4:n funktioista ja pinnan määritelmästä. Näin tulisi ajatella, jos R^8:n oletetaan olevan aina olemassa ja kaikki muu on sen alimonistoa, mutta tämä tarkoittaa juuri sitä, että esim. yksi p-adinen avaruus on R^2:n (R^1:n?) alimonisto ja tehty R^n -> R kuvauksella, vaikka kuvaus R -> Q_p -> R jossain muualla pitäisi niitä edes kuntana.

    Ylempi monisto ja avaruus pelkän karteesisen tulon tai yhteisen Hilbertin avaruuden asemasta vaikuttaa algebroja enemmän, siihen että R^8 -avaruudella määritellään aliavaruuden kaarevuus. Tällä sivulla lukee jossain muussa tekstiosuudessa, että CP-tapauksessa TGD:stä sillä on joka R^4 pisteessä myös CP2-avaruus, mikä voi tarkoittaa tensoritulon projektio-H:ta, missä kvanttiobjektin puhtaat tilat ovat puolijanoja ja yleisesti mikä tahansa funktionaali. Vaikka () x CP2 olisi vain tällainen Hilbert-versio, johon tehdään kuvaus R^8:sta, on olemassa kaareva C^2 alimonisto, joka ei ole tavallinen aika-avaruus, ja jonka dynaaminen kaarevuus (vakio?) tuottaa aina eri kuvauksen esim. samalle objektin alkutilalle ja aikavälille. Jos taas projektioavaruus on leikkauksella muodostettu avaruus isäntäavaruudesta, niillä on molemmilla kaarevuus, ja nämä ovat R^8:n tai C^4:n alimonistoja. Muutoin, jos CP:tä käytetään vain SM-symmetrioiden vuoksi, voidaan unohtaa molemmat tavat käyttää R^8:aa ja ohittaa ongelmia, joita tulee 8D-ulotteisesta gravitaatiosta, missä kaikkien ylimääräisten ulottuvuuksien pitää kaareutua sulkemaan avaruutta lähelle R^4:ää (ilman hyvää dynaamista syytä, vaikka dynaaminen syy antaa vakion, kuten tässä aina?) mutta miten jokin muu TGD tuli koskaan toimeen, ja mitä nämä muut algebralliset laajennukset ovat verrattuna symmetrioihin, tai jos ne eivät ole samalla kaarevia?

    >>>>>>>>>>> [MP] Tämä on nollaenergiaontologiaan perustuvan tietoisuuden teorian ennustus. En lähde liikkeelle mistään itämaisesta filosofiasta, jonka yrittäisin reprodusoida. Tämä kahtiajako vastaa sitä, etttä kvanttihyppyjen sarjassa CD:n passiiviseen reunaan liittyvä tila ei muutu: Zeno efektin analogia. Aktiiviseen reunaan liittyvä tila muuttuu jokaisessa unitaarisen evoluution vastineessa jota seuraa ”heikko” mittaus. Tämä vastaa aistisyötettä. Iso (”tavallinen”) kvanttihyppy merkitsee ”itsen” kuolemaa ja reinkarnaatiota vastakkaisessa ajan suunnassa. Nyt aktiivisesta CD:n reunasta tulee passiivinen ja päinvastoin. Tälle antaa suoraa tukea Minev et al havainnot kuten myös Libet.
    [MP] ”Itsen kehitys” on analogia Zeno efektille. Nolla-energiaontologian tilat ovat pareja tavallisista 3-D tiloistan CD:n reunoilla. CD:n passiivisella reunalla tila ei muutu – Zeno efekti. Aktiivisella reunalla, jossa tapahtuu heikkoja mittauksia se muuttuu ja se vastaa ”itsen” aistihavaintoja.>>>>>>>>>>>

    Rinnastetaanko vain passiivinen reuna ns. gravitonien tai gravitaatiokentän aiheuttamaan tilan romahtamiseen? Kun esim. Bohrin ratkaisussa (n=0) elektronin paikka ei koe sellaista vuorovaikutusta, että radan pitäisi muuttua pisteeksi, sen ja gravitaation systeemi on varmasti joko erilainen kuin on sähkömagneettisesti mitatulla elektronilla tai sitten gravitaatio romahduttaa hyvin pienen joukon systeemeistä eikä välttämättä yhtään paikkamuuttujaa. Kaikilta näiltä teorioilta voi tällöin kysyä, tiedetäänkö näistä systeemeistä tällöin jotain, ja TGD:ltä kysyn, onko TGD:n passivinen reuna varmasti sellainen, että siellä tämä vaikutus esiintyy muuttei muualla ja onko sen määrittelemä tila ja sen herkkyys samanlainen kuin muissa teorioissa. Ensin esimerkiksi se, että päteekö romahdus kaikille tiloille ja suureille (tämä voi johtaa myös epäkommutatiivisuuteen ja planckin rajaan, mutta gravitaation/avaruuden rooli ja siihen kytkeytyminen kiinnostaa enemmän)?

    Moniin kenttäteorioiden esim. Feynman-sovelluksiin verrattuna pelkkä valokartion pitäminen ainoana kausaalisena osana on liian rajallista, eikä yleiselle objektille, jonka paikka ja liikemäärä ovat alussa kvanttitilassa, voida antaa yhtä valokartiota. Vaikka voitaisiin, täytyisi ainakin integroida tapaukset, joissa mikä tahansa muu yhdistetyn systeemin objekti esiintyy poissa massakuorelta ja tässä tapauksessa sellaiset alkuobjektit, joiden paikan jne. keskiarvot esiintyvät ensimmäisen objektin valokartion ulkopuolelta.

    Minev ei palauta tilaa kääntämällä ajan suuntaa. Hänellä saattaa olla kaikissa liikeyhtälöissä ajankääntö-symmetria, mutta tämä on voinut olla sivuseikka eikä ole se, mikä on johtanut eri asetuksiin. Kun Minev nimittäin kääntää tilan evoluution, hän vaihtaa koko evoluutiooperaattorin käyttämällä uusia asetuksia ulkoisille kentille. Tällöin systeemi A, joka pysäytetään ei sisällä mitään samoja todennäköisyyksiä lopputilalle kuin romahtamista jatkava systeemi B, vaikka teoriassa molempien vektoritilojen kannat ovat samat (A ja B tehdään aina eri hiukkasille, jolloin maailman Hilbertin avaruudessa kannat eivät ole samat). A:ta ja B:tä kuitenkin verrataan keskenään ja B:n tiloja pidetään jonain A:n muuna vaihtoehtotulevaisuutena, vaikka näin ei ole.

    Entä jos Minev antaa ison kuolemahypyn tapahtua, ja tämä hyppy kestää kauan, ja timantin alareunasta vain osio, joka on t > dt yläpuolella, on aktiivinen tästä eteenpäin, niin mitä gravitatiivinen vaikutus näille reunan osille sitten tekee (Minev on mm. gravitaatio, mutta jatkaako hän)? Entä jos Minev ei anna hypyn tapahtua, mutta saavuttaa vain pienemmät todennäköisyyserot kuin 0.8-0.2 (hänellä alkutila) kuvassa 4b, niin eikö reunan aktiivisuuskin voi olla superpositio-tilassa?

    >>>>>>>>>>> [MP] Ei ihan näin. Kytkentävakioita ei imetä h:n.>>>>>>>>>>>

    Jos h kasvaa ja kytkentävakiot pysyvät vakiona, tiedetään että vapaan hiukkasen paikka-aalto on leveämpi. Aaltopaketit leviävät ajassa nopeammin suuremmalla h:lla. Tämä vapaan hiukkasen paikan todennäköisyyden diffuusio on tällä sivulla:
    https://quantummechanics.ucsd.edu/ph130a/130_notes/node83.html
    Systeemien ominaisenergiat kasvavat, ja varmaan esim. hiukkanen laatikossa ei toteutuisi leveälle aallolle ilman vastaavaa suurempaa potentiaalia.

    Pelkästään h:sta riippuva käytös on mahdollisesti peräisin vain ensimmäisiin polkuintegraali-metodeihin johtaneesta kvanttivaikutuksesta ja siitä johdetusta mekaniikasta, joka perustui silloin klassisen Lagrangen funktion vaikutukseen ja sen poikkeuksiin, jolloin poikkeusten häviämistä on voitu pitää klassisena rajana. Ja näitä häviämisten sijaan useammin tutkittiin vain h:n nollan lähestymistä ternissä e^(iS/h).
    https://arxiv.org/abs/1201.0150
    Artikkelissa muistutetaan kovasti, että raja vastaa silloinkin probabilistisia liikeyhtälöitä (ehkä rho:kin on diagonaalinen), mutta siinä on yksi tapaus (9) pienelle joukolle kvanttitiloja, jossa näkyy h:n osuus verrattuna osuteen ilman h:ta. Myös lähtökohdan ongelmia selvitetään sivulla 9-10.

    https://en.wikipedia.org/wiki/Classical_limit
    Sanoo periaattessa, että kvanttioskillaattori (jolle on annettu klassinen energia) muuttuu klassiseksi, joissakin korkeampien energioiden ominaistiloissa. Jos suuremmalla energialla absoluuttisesti olisi oikeasti merkitystä systeemin kvanttiluonteen pitämiseksi, tätä kompensoi se, että energiat (ei klassisen vaan kvanttitilan) ovat jo suurempia ja kvanttiluku n määrittäisi klassisen rajan ilman h:ta. Energiaa esiintyy myös yksittäisen ison h:n muodostamassa suuressa irrallisessa populaatiossa jatkuvasti, koska suuret energiavälit muodostavat korkeampienergisiä bosoneja, joita koko systeemi joutuu vaihtamaan. Pelkkä bosonin emittointi johtaa klassisuuden kasvamiseen ja voi olla verrannollinen energiaan ja muutos on paikkatilalle lopulta itsenäinen h:sta.

    Jos tuo yksi malli kuitenkin unohdetaan eikä pidetä yksittäisen systeemin klassisuuden muutosta edes mahdollisena täydellisessä vakuumissa, niin kvanttiluonto häviää vain vuorovaikutuksissa, mitä voi tulla objektille enemmän johtuen sen ja muiden objektien paikkatilojen kasvavasta päällekkäisyydestä, mitä objektit varmaan kompensoivat olemalla harvemmassa pelkkien törmäysten avulla (aine on hetken aikaa klassisempaa) tai jopa mudostamalla avaruuteen suuremman osan bosonista ainetta ja laajentamalla sitäkin nopemmin kuin pienellä h:lla (sanoit tosin jotain massan laskemisesta ja ehkä uusiksi?). Ja vuorovaikutukset mitä näin tulee pysyvät samoina tai hidastuvat.

    Esim. Minevin artikkelissa ei sanota, mutta h esintyy /Omegoiden nimittäjissä ja tavallisesti myös radiaatio-kertoimissa (spontaani emissio), nimittäjissä, ja tilat, romahtamiset ja pakittamiset ovat esim. tanh(x/h), eli suuremman h:n tila kehittyy hitaammin (”dekoherentissa ajossa” h:ta ei ole). Kaikissa muissakin unitaarisissa kehityksissä tai melkein kaikille energioille, jotka ovat h-verrannollisia, aikakehitysopraattori on h:sta vapaa, ja h ei vaikuta hetken t tilojen todennäköisyyksiin. Objektien toisiinsa vaikuttaminen ei siis muutu eikä edes tajuus oman esimerkkini a-oskilloinneissa muutu. Se tarkoittaa ettei kietoutuminenkaan muutu niissä samaan perustuvissa tapauksissa, mitä kietoutumisen unitaarisesta mudostumisesta tiedetään.

    Tykkää

    1. Jos tuntuu siltä, että näistä posteista tulee liian pitkiä niin voi pohtia mahdollisuutta että käymme keskustelua myös emailitse. Oma osoitteeni on matpitka6@gmail.com. Seuraavassa käyn läpi kommenttisi jokseenkin kokonaisuudessaan.
      Olen merkinyt sinun puheenvuorosi alkamisen %%:llä ja omani [MP]:llä.

      *****

      >>>>>>>>>>> Näyttäisi siltä, että kirjoittaja uskoo aaltofunktion dekoherenssin käsitteeseen joka on yksi yritys tulkitsemalla pelastaa kvanttimittausteoria. Näitä pelastusyrityksiä on valtaisa määrä ja näyttää vahvasti siltä että potilas on kuollut ajat sitten.
      >>>>>>>>>>>

      %%
      Dekoherenssilla olen tarkoittanut tilojen romahtamista deterministisesti ja paljon tai vähän jossain ympäristössä, mikä ei ole sanan ensimmäinen määritelmä. Pelkän (de)koherenssin evoluutio on täydelle dekoherenssille sellainen, että puhtaasta tilasta tulee diagonaalinen tiheysmatriisi ja kvanntikorrelaatioista klassisia korrelaatioita, jotka ideaalisesti eivät myöskään painotu mitenkään uudella tavalla ja mahdolliset mittaustulokset jäävät kokonaan muuttumatta.

      Jokainen yksittäinen malli jollekin evoluutiolle kuvaa tämän dekoherenssin kyllä täysin teoriassa ja hyvin, koska systeemit voivat olla yksinkertaista muotoa, ja moni malli varmasti on myös joissain kokeissa vastaavasti olemassa (mutta täytyy todistaa havaitsemalla koherenssi). Moni kvanttiobjektin ympäristöön kytkevä malli, joka muuttaa myös mittaustodennäköisyyksiä deterministisesti kohti romahdusta perustuu samoihin ideoihin kuin niitä edeltäneet mallit. Tai jos olet oikeassa, sitten on todennäköisesti käynyt niin, että vasta kun dekoherenssille piti saada malli, joka perustuu oikeisiin vuorovaikutuksiin, niin se ei ollut enää pelkkää dekoherenssia. Ainoa ideologinen seuraus dekoherenssisysteemeistä tapahtui silloin kun ne varmaan ensin tulivat ja käsitteenä sen nimi on ”many worlds theory”.

      [MP] Dekoherenssi on mahdoton unitaarisena aikaevoluutiona jos systeemi on suljettu. Tiheysmatriisin ominaisarvot säilyvät ja myös siihen liittyvä entropia. Puhtaalle tilalle entropia on nolla ja lopputilalle nollasta poikkeava, jos dekoherenssi tapahtuu.

      Avoimelle systeemille tilanne on toinen. Jos systeemi kietoutuu aikakehityksen aikana ulkomaailman kanssa niin tätä kietoutumista kuvaa tiheysmatriisi ja se yleensä tulee ei-diagonaaliseksi ja entrooppiseksi. Yksi tapa kuvata termalisoitumista on tämä mutta kvanttikietoutuminen ei ole vielä termalisoitumista tai dekoherenssia.

      Tilastollinen kuvailu edellyttää systeemin kopioista muodostettua ensembleaä jolla on sama vuorovaikutus ympäristön kanssa. Kun mittaus on suoritettu miin lopputilat ovat tämän tiheysmatriisin ominaistiloja tiheysmatriisista yksittäisen systeemin kvanttikietoutumisen kuvaaja tuleekin tiheysmatriisi ensemblelle.

      Oma näkemykseni on että tilafunktion reduktio on täysin universaali prosessi eikä vain inhimillisen havaitsijan suorittama. Tiheysmatriisi kvanttikietoutumiselle on universaali observaabeli.

      >>>>>>>>>>> TGD:n inspiroima tietoisuuden teoria palauttaa vapaan tahdon tilafunktion reduktioon ja samalla ratkaisee kvanttimittausteorian perusongelman.
      Esimerkiksi Minev et al:n tekemät kokeet atomifysiikan tasolla antavat suoraa tukea varsi dramaattisille ennustuksille.
      Samoin Libetin havainnot.

      >>>>>>>>>>>

      Minevin ”To catch and reverse a quantum jump mid-flight” (2019, arxiv: 1803.00545)

      on kuin mikä tahansa mainituista todennäköisyyksiä muuttavasta systeemeistä. Yhtä Supplementaryssä olevista malleista kutsutaan koherentiksi, koska sen aikakehitys voidaan antaa ket-vektoreille ja suurten kenttien/kytkentöjen approksimaatiossa näiden kertoimet toteuttavat täyden mittaustapahtuman (mutta suljetussa muodossa ei voitu osoittaa, että aina). Tämä ei ole kaikkein realistisin vaihtoehto vaan artikkelia tehdään myös applikaatioille, joissa halutaan vähentää koherenssin häviämistä ympäristöön systeemeissä, joissa näin käy, ja aina ennen mittausta, koska sitä pidetään virheenä. Kun seuraava malli ei tuota täyttä mittausta, tätä ei kannata pitää sen ”inkoherenttisuuden” syynä. Päätekstissä esim. s.5 koherentiksi kuvataan mahdollista löytynyttä mittausteoriaa, jolla on ensin oltava evoluutio, minkä yhteydessä kööpenhaminan tulkintaa ei ikäänkuin pidetä koherenttina, mutta kööpenhamina tai von Neumannkin päätyy täyteen mittaukseen.

      [MP] En jaksa uskoa havaitsijoiden esittämään teoriaan havaintojensa selityksenä vaan pointtini on se, että tarvitaan uusi ontologia havaintojen selitykseen. Lähtökohtanani ovat havainnot, ei niiden selitys. Epäjatkuvan kvanttihypyn sijasta havaitaankin klassinen determistinen evoluutio joka johtaa johonkin lopputiloista. Tässä nolla-energia ontologia tulee avuksi. Myöskin kun evoluutio on alkanut häiriöt eivät enää vaikuta: tässä on tulkinta se että kvanttihyppy on jo tapahtunut ja tarkastellaankin deterministixtä aikaevoluutiota joka johtaa käännetyssä ajansuunnassa lopputilasta geometriseen menneisyyteen.

      >>>>>>>>>>> [MP] Tässä on taas massiivinen väärinkäsitys kyseessä. Kirjoittaja lähtee omista oletuksistaan joiden mukaan klassinen fysiikka on kvanttifysiikan raja pitkissä pituus-skaaloissa. TGD:ssä (nolla-energia-ontologiassa) tilanne on täysin toinen. Kvanttitilat ovat superpositioita klassisista deterministisistä aikakehityksistä: 3-D aika=vakio snapshot kvanttitilana on korvattu superpositiona 4-D avaruusaikalehdistä kausaalitimantin sisällä..

      >>>>>>>>>>>

      %%
      Jos tietoisuuden pienimmän h:n ja kehon osa on kvanttitiloissa, niin sitten on vain kvanttitilat, jotka mittaavat kvanttitiloja. Mutta näiden mittaamaa informaatiota on vaikeampi säilyttää ja muodostaa muistia mittauksista. Kvanttimuiston uudelleen tiedostaminen kvanttitietoisuudella jne.

      En ole pitänyt pituuksia minään, vaan sitä että joskus kvanttiobjektille tulee enemmän kvanttiobjekteja vastaan. Mitään muuta selitystä ei ole sille, että omankin selityksesi pitää sitten selittää miksei kaikessa aineessa ole olemassa montaa determinististä aikakehitystä kerralla? Vosiko niiden määränkin selittää deterministisesti? Ei silti kannata ajatella tämän antamaa tulosta (ykköstä) mekaniikkojen rajana.

      Kuinka monta dimensiota on yhdessä lehdessä, maksimidimensioisen CD:n tapauksessa? Koska jos niitä on kolme avaruudellista dimensiota, niin superpositiossa on snapshotteja eikä aikakehityksiä ainakaan tämän avaruuden ajalla. Yhdistetty systeemi on varmaan snapshot eikä 4D-tilojen tensoritulo, vaikka jälkimmäistä ei voi estää?

      [MP] Nolla-energia-tila on superpositio determinististä klassisista aikakehityksistä jotka johtavat toisistaan poikkeavista alkutiloista lopputiloihin. Tämä on aaltopaketin vastine. Lisäksi ennustuu kausaalitimanttien hierarkia upotusavaruustasolla (H=M^4xCP_2) ja avaruusaikalehtien hierarkia avaruus-aikatasolla.

      *Hierarkioista: Pinnalle kondensoitunut pienempi avaruusaikalehti – topologinen summa johon liittyy ääärimmäisen pieniä madonreikäkontakteja – on alisysteeminen vastine. Nämä pinnat pinnoilla muodostavat avaruusaikalehtien skaalahierarkian, samoin niihin liittyvät CD:t. Kenttäteoria-approksimaatiossa nämä avaruusaikalehdet korvataan M^4:n alueella ja saadaan Einstein-YM teoria.

      *Tietoinen havaitsija identifioituna sarjana ”heikkojen” kvanttimittausten sarjana jossa havaitsijan CD kasvaa tilastollisessamielessä niin, että CD:n toinen reuna ja tilaparien jäsenet sillä eivät muutu (Zeno efekti) mutta toinen CD:n reuna siirtyy kauemmaksi ja tilat sillä muuttuvat on havaitsijan, ”itsen” vastine.

      *Avaruusaikalehden dimensio on 4. CD:n dimensio on 8. CD=cd xCP_2, missä M^4 kausaalitimantti cd on tulevaisuuteen ja menneisyyteen suuntautuvien valokartioiden leikkaus: salmikki on hyvä 2-D visuaalisaatio.
      Superpositio on avaruusaikapinnoista CD:ssä, jotka yhdistävät 3-pintoja CD:n valokaltaisia reunoja. Näihin 3-pintoihin molemmissa päissä liitetään kvanttitilat tavallisten kvanttilojen vastineina ja niitä yhdistää deterministinen aikaevoluuto. Tila ZEOssa on superpositio näiden 3-D tilojen pareista. Tai sitten niitä yhdistävistä deterministisistä aikaevoluutioista: tämä realisoi hologrian/Bohrin orbitologian.

      Jos determinismi aikakehitykselle on eksakti niin tila jommassa kummassa päässää määrää tilan toisessa päässä. Tarkalleen näin ei näytä olevan. Saadaan täydellisen determinismin rikko ja M^8 kuvassa klassinen aikakehitys liimataan äärellisen monesta palasta jotka vastaavat diskreettejä M^4-ajan arvoja t=r_n. Hupaisaa on analogia säiemalleihin. Näihin liitoskohtiin t=r_n:ssa liittyy 6-pallo erikoisratkaisuna algebrallisiin yhtälöihin jotka yleensä antavat neli-pinnan. 6-pallon on kuin braani jossa avaruusaikapinnat liittyvät toisiinsa.

      Ne ennustuvat M^8-kuvasta ja t=r_n vastaa avaruusaikapinnan määräävän reaalipolynomin juurta. Äärellinen summa näiden ei ihan determinististen aikakehitysten yli on Feynmanin polkuintegraalin vastine. Kun polynomian aste menee äärettömiin ja juuret tulevat miltei jatkumoksi, saadaan jotain joka vastaa polkuintegraalia.

      Vielä työn alla olevasta artikkelista löytyy enemmän. Erityisesti unitaarisen S-matriisin korvaamisesta Hilbert-kimpun Kahler geometrialla, joka on tämän avaruuden sisäinen ominaisuus sen sijaan että se olisi ad hoc valinta unitaariselle aikakehitykselle äärettömän monien joukosta.

      http://tgdtheory.fi/public_html/articles/kahlersm.pdf .

      >>>>>>>>>>>
      [MP] Kompleksiluvut ovat rationaalinen täydentymä jossa algebralliset luvut vielä täydennetään jatkumoksi. Mitä tulee havaitsemaamme fysiikkaan, emme voi tietenkään olettaa että se vastaisi tätä ideaalista rajaa. On kyse myöskin havaitsijan kuvaamisesta, ei minään ideaalisena muukalaisena vaan osana systeemiä. Kognitiiviset esityksen ja laajennusten hierarkia evoluutiohierarkiana tulevat tällöin mukaan.
      >>>>>>>>>>>

      %%
      Kompleksiluvuista ja havaitusta fysiikasta voi sanoa sen juuri, että todellinen maailma ei niitä välttämättä kaipaa. Se käyttää vain osaa ns. algebraryhmistä ja teoriat, joissa on ylimääräisiä ryhmiä, joutuvat hankkiutumaan niistä eroon kussakin tapauksessa. Nimenomaan se että kompleksiluvuilla on suljettu algebra, kun suljettavuus ei ole vaadittu ominaisuus (tätä ei tiedetä, jos se kuitenkin on pakollista kaikenteoriateorioissa, jotka on jo kiinnitetty niin fyysisiksi kuin voi), voi olla epäfyysistä siksi ettei maailmassa ole negatiivisia lukuja ja kaikki reaalifysiikka käyttää reaalilukuja myös jonkinlaisena valittuna mittana.

      Ajatusharppauksesi ei voi tarkoittaa, että luvuista päädyttäisiin mittaamisprosessiin, koska osana systeemiä oleminen on fyysistä todellisuutta ja jää jäljelle vaikka esim. ideaalinen kompleksilku-teoria sen esittäisi. Saman esityksen muuttuminen eri luvuiksi tai ryhmiksi ei vaikuta suoraan siihen, onko esityksessä mukana näitä systeemin osia.

      [MP] Ei kvanttiteoria tule toimeen ilman kompleksilukuja. Ne ovat aivan olennainen osa kvanttiteoriaa. Esimerkiksi oskillaattorioperaattoriformalismi edellyttää kompleksifikaatiota Lie-algbralle, muuten hävitys- ja luomisoperaattoreita ei voida määritellä.

      Mittausprosessia tilafunktion reduktiota ei ole tarkoitus redusoita mihinkään – ei myöskään lukuteoriaan – vaan hyväksytään se faktana ja kysytään miten ontologisia uskomuksia on muutettava, jotta vältytään ristiriidoilta, joita sitten yritetään tulkita maton alle. Nolla-energiaontologia (ZEO) on vastaus haasteeseen.

      Lukuteoriaan päädytään kun kysytään mitä on kognitio: mitä ovat ajatusten matemaattisfysikaaliset korrelaatit. Reaaliluvut kuvaavat aistihavaintoja. p-Adiset lukukunnat tulevat mukaan luonnollisesti kognition ja intention korrelaatteina ja niiden mukana rationaalilukujen laajennukset ja adelinen fysiikka jossa fuusioida reaalifysiikka aistihavainnolle ja p-adiset fysiikat kognitiolle.

      >>>>>>>>>>>
      Tällä tavalla klassinen teoria- avaruusaikapintojen geometrodynamiikka – ja myös Diracin yhtälön yleistys – ovat eksakti osa teoriaa.Klassinen teoria ei ole todellakaan approksimaatio. Kvanttihypyt tapahtuvat näiden superpositioiden välillä. Klassinen deterministinen kausaalisuus pätee eikä johda ristiriitaan kvanttihypyn ei-deterministisyyden kanssa. Tämä on koko teorian kuningasidea.
      >>>>>>>>>>>

      %%
      Minevin artikkeli ja muutkin yrittävät osoittaa kvanttihyppyjen olevan deterministisiä. Hänen teoriassaan on 3D Galilein avaruus. Deterministisessä kvanttihypyssä odotetaan aika dt >> 0, ja lineaarikombinaatiossa (vektorien tai tiheysmatriisien) ei ole jäljellä enää useita tiloja. Voitko nimittää mainitsemiasi hyppyjä tästedes 4D-hypyiksi, jos niistä nimittäin tulee joko dt=0 -rajalla tai mieluiten Galilein-approksimaatiossa samanlaisia deterministisiä 3D-hyppyjä?

      Jos TGD on geometrodynamiikkaaa, sen 3D-lehtiavaruudet olisivat metriikaltaan indusoituja 4D-avaruudesta.

      [MP] Kuten totesin, en ota vakavissaan Minev et alin teoriaa, enkä kvanttihyppyjen determinisyytä vaan heidän havaintonsa. Kvanttihypyt eivät ole deterministisiä TGD:ssä. Kvanttilat ovat determinististen aikakehitysten superpositioita ja ”isossa” kvanttihypyssä syntyyn aikakäännettyjen kehitysten superpositio, joka johtaa lopputilasta geometriseen menneisyyteen. Tämä superpositio (tai sen keskiarvo) havaitaan ja voidaan päätä illuusioon, että kvanttihyppy on deterministinen.

      TGD kuva ratkaisee myös kysymyksen siitä miksi maailman näyttää klassiselta ja determiniseltä. Tavallisesti uskotaan että näin on pitkissä skaaloissa. ZEOn ennustus on että ulkoiselle havaitsijalla kvanttihyppy näyttää aina klassiselta aikakehitykseltä ja että maailma on kvanttimaailma kaikissa skaaloissa. Libetin kokeet on yksi sovellutus. Maanjäristysten aika-anolioiden selitys on toinen sovellutus: http://tgdtheory.fi/public_html/articles/earthquarks.pdf .

      4-pintojen metriikka voidaan indusoida suoraan H:ta tai sitten nelipinnalta. Lopputulos on sama.

      >>>>>>>>>>> [MP] Ei-relativistisen kvanttifysiikan kuvailu perustuu Hamiltonin käsitteeseen. Se on ok niin kauan kuin Newtonilainen kuva on hyvä approksimaatio ja varmastinen hyvä fenomeologinen työkalu yksinkertaisten systeemien kuvailuun. Siitä ei ole kuitenkaan apua TGDssä.

      Pistemäinen hiukkanen korvataan 3-pinnalla, jonka rata määrittelee avaruusaikapinnan alueen. Säiemallin 1-D säie korvautuu 3-D pinnalla. Vaati kuutisen vuotta vakuuttautua siitä, että kvanttikenttäteorioiden polkuintegraali ei todellakaan toimi. Lopputulos oli käsite ”klassisten maailmojen maailma” avaruusaikapintojen muodostamana ääretön-D Kaehler geometriana, joka pelkästä olemassaolostaan on uniikki jo luuppiavaruuksien tapauksessa, plus ”nolla-energia-ontologia” joka korvaa Newtonilaisen ontologian. Siinä yleinen koordinaatti-invarianssi pakottaa identifoimaan kvanttitilat superpositioina avaruusaikapinnoista jotka ovat analogisia Bohrin radoille.
      >>>>>>>>>>>

      3D-kehitykset, joiden avaruuksien on sanottu nauhoissa olleen ei-kaarevia, määrittävät kuitenkin yllä jokaisen TGD:n ratkaisun ja lopputilan, ja TGD lisää vain niiden probabiliteetit, ja osan systeemin dynamiikasta siinä mielessä, että superpositiotila muuttuu hypyn aikana?

      [MP] 3-D pinnat- ei kehitykset- ja niiden aikakehitykset 4-pintoina. Näiden nelipintojan kvanttisuperposiot nollaenergiatiloina. Kaikki nämä pinnat ovat kaarevia.

      %%
      Jos 4D-avaruudessa on avaruudenkaltaisia lehtiä, ja niiden kaarevuus ei muutu lehtien välillä, kun aika muuttuu, niin avaruuden puolesta systeemissä on mahdollista olla millä tahansa hiukkasta ja timanttia ylemmällä tasolla säilyviä suureita ja Hamiltonin mekaniikassa on jopa paljon järkeä, jos suureille ei tapahtuisi mitään muutoksia 4D-superpositioissa (ehkä tämän pitää seurata TGD:n Galilei-approksimaatiosta), joka estää nimenomaan kvanttiobjekteja olemasta esim. energialtaan säilyviä.

      |MP] Säilyvät suureet ja niiden kvanttioperaattori-vastineet fermioneille ovat summia kolmipintoihin ja niihin liittyvien partonipintojen säikeiden kontribuutioista jotka eivät säily erikseen.

      Avaruuden kaltaisia avaruusaikapinnan alueita todellakin ennustuu: ne vastaavat madonreikäkontakteja kahden avaruusaikalehden välillä. ja CP_2 : n palnen tai sen kanssa isometrinen ”warped” pinta on hyvä lähtöapproksimaatio. Niiden M^4 projektio on valonkaltainen geodeettinen viiva ja tulkinta on alkeishiukkasten geometrisina vastina. Koko on CP_2:n koko-skaalan luokkaa.

      %%
      Et ole kertonut miten 4D-superpositio muuttuu tai tulkitaan muuten kuin pitämällä sitä epädeterministisenä, mikä yleensä tarkoittaa kaikkien invarianssien puuttumista objektin suureista ja yhtälöistä, koska yhtälöjä ei hypyn aikana ole.

      [MP] 4-D superpositio korvautuu uudella sekä pienissä että isoissa reduktioissa (”heikot” ja tavalliset reduktiot).

      Pienessä reduktiossa CD:n koko kasvaa tilastollisessa mielessä ja avaruusaikapintojen superpositio sen sisällä muuttuu:

      olen mallintanut tätä prosessia adiabaattisena aikakehitysenä. Aikakehityksen kesto siis muuttuu. Aikakehitysten alkuarvot CD:n passivisella eivät muutu: kolmipinat siellä säilyvät samoina kuten myös monihiukkastilat. Tämä vastaa Zeno efektiä. Se että CD:n aktiivinen reuna siirtyy kauemmas pasiivisesta antaa vaaditun korrelaation subjektiivisen ajan (kvanttihyppyjen sarja) ja geometrisen ajan (CD:n kärkien väli välillä).

      %%
      Koordinaatti-invarianssia ei voi pakottaa millekään paitsi havainnoille, joista ollaan erittäin varmoja. Onko tässä oltava varma joistain tavallisen kvanttimekaniikan arvon tai yhtälön invariansseista? Miten se on johtanut kokonaan omaan romahtamisdynamiikkaan? Jossain näin, että oli tehty koordinaattimuutoksista myös kvanttitiloja ja suureita, jolloin kokonaistila oli yksi ja sama superpositio näiden kanssa. Ja muunnoksilla oli Hilbertavaruus ja Hamiltonilainen kehitys. Tämä vastaa hyvin paljon mittakenttien lisäämistä hiukkastilaan.

      [MP] Kvanttihypy ei ole deterministinen aikakehitys TGDssä. S-matriisin vastine koodaa transitio-amplitudit ja omaa WCW:n supersymplektiset symmetriat.

      Yleinen koordinaatti invarianssi (GCI) on empirian kanssa sopusoinnussa ja geometrisoinen perusoletus, jota voi pitää läheiset triviaalina. Tämä ei merkitse sitä etteikö olisi erikoisasemassa olevia koordinaatistoja. Esimerkiksi H:n koordinaatit antavat avaruusaikapinnalle sellaisia – etenkin Minkowskikoordinaatit. TGD:ssä GCI implikoi holografian ts. se että 3-D pinnat ovat kausaalideterminantteja. Itse asiassa saadaan vahva holografia: partoniset pinnat ja 2-D säieradat ovat kausaaliodeterminantteja ja määräävät nelipinnan.

      %%
      Kvanttigravitaatiot, paitsi säieteoria, ovat yleistetyn Hamiltonin mekaniikan teorioita, missä voi olla 3D Hamiltonin funkiton asemasta 4D-funktioita ja Hamiltonin-ehtoja. Idea siitä, että gravitaatiolle yleisen 4D-avaruuden sijaan riittää tarkastella (3+1)D avaruuksia liittyy siihen, että on kaksi kaareutumisen vapausastetta, joiden parametrisoitu evoluutio voi olla 3-ulotteisen metriikan (max. 3 vapaata muuttujaa jo tehdyn koordinaattien kiinnityksen jälkeen) evoluutio. Näihin kahteen asteeseen päätyminen ei ole välttämättä uniikkia, mutta täysin ekvivalenttia sen kanssa, että ne seuraavat yleistetystä Hamiltonin mekaniikasta, missä alkuehdot monissa tarkasteluissa näyttävät siltä kuin lehditys olisi tehty jo (ja kaikki tämä ajattelu kiertää kehää), mutta menetelmän pitäisi tarvita vain koordinaattimuuttujia, joista jotain pidetään ajan parametrina. Jos TGD ja kvanttigravitaatiot ovat molemmat totta, niin silloin TGD:ssäkin on enemmän H:n kaltaisia formalismeja löydettävissä (säieteoriaa ja H:hon perustuvia teorioita on osoitettu samoiksi ainakin ilman gravitaatiota).

      [MP] En ole ihan varma mitä tarkoitat 4-D Hamiltonilla. Kyllä yleisen suhteellisuusteorian kvantitusyrityksissä lähdettiin liikkeelle 3-D pinnoista kausaalideterminanttina. Hamiltonille kävi kuitenkin nolosti: yleisen koordinaatti-invarianssin vuoksi se hävisi joten sitä ei voitu pitää observaabelina ja unitaarisen aikakehityksen generoijana.

      Otan faktana, että 3-D pinnat ovat kausaalideterminantteja loistavana approksimaation – 4-D Hamiltonit vastaisivat 4-D:ssä satunnaisuutta ja mukaan tulisi aika viidentenä koordinaatina. Subjektiivinen aika kvanttihyppyjen sarjaa ei ole toinen koordinaatti ja korreloi vahvasti geometrisen ajan kanssa. Kun koet erilaisia kokemuksia ja katsot aina kelloa,huomaat että sen lukema kasvaa.

      %%
      Jos 3-pintainen kvanttiobjekti lähestyy vapaan hiukkasen aaltofunktiota, sen aaltopaketilla ja keskiarvolla voi olla rata, mutta aaltoa ei voi määritellä kuin äärettömässä alueessa. Vapaan hiukkasem aaltofunktio ei ole vakio kuten Bohrin rata, eikä mitään t:stä riippuvaa pintaa voi enää pitää samanlaisena kuin avaruudenkaltaiset lehdet.

      [MP] Kyllä aaltoja voi määritellä vaikkapa laatikossa. Bohrin rata käsitteenä edelsi Schrödingerin yhtälöä ja antoi yllättävän hyviä ennustuksia. Polkuintegraalissa Bohrin radan vastineen voisi ajatelle stationaarisen vaiheen approksimaatioksi, jossa vaikutus kehitetään maksiminsa ympärillä ja tehdään Gaussinen approksimaatio.

      %%
      Rataa joka 4D:hen tulee voidaan aina esittää jokaista aikaa vastaavina avaruuslehtinä, joista jokaisessa on objekti, mutta tällöin jokaisen lehden määritelmä on se, minkä se saa siivuteltavana olevasta 4D-avaruudesta. Muutenkaan tältä ei pitäisi pystyä välttymään, koska jos yrität silti tehdä makroavaruuteen toisenlaisia aliavaruuslehtiä jonkin objektin radasta (tangentin suuntaan parametrisoiden tms.) on tämä rata aina aluksi määritelty kyseisessä makroavaruudessa, eikä sen aliavaruudet ole erillisiä määritelmiä jossa ne ovat esim. klassisia. Muutkin käyttävät lehtiä joskus kuin niitä voisi käyttää lokaalisti eli vaikka 4D-avaruuden kaarevutta ei kokonaan tunneta joka pisteessä, mallissa on tällöin samoja heikkouksia kuin tavallisessa kvanttimekaniikassa, joka ottaa vain vähän ympäristöä huomioon.

      [MP] En asettele mitään objekteja avaruusaikalehdille.

      *Avaruusaikapinnat on klassinen tosiolevainen. Lehdille voi toki olla topologisesti kondensoituneena pienempiä lehtiä ja ne me miellämme fysikaalisina objekteina joita näemme ympärillämme ja mikroskoopissakin. Klassinen aine ”substanssina” katoaa geometriaan ja topologiaan TGDssä.

      *Saatttaa olla, että tarkoitat avaruusaikalehdillä jotain muuta kuin minä. Ideana on että avaruusaikapinnalla on suuri määrä lehtiä, joilla on enemmän tai vähemmän *sama projektio* M^4:ään. Näiden lehtien etäisyys on CP_2:n skaalan luokkaa ja siis äärimmäisen pieni ja kenttäteoriarajalla ne approksimoidaan yhdellä M^4 alueella. Voi ajatella lähes-tasopintoja äärimmäisen ohuen lasilevy sisällä. Ne voivat koskettaa toisia – madonreikäkontaktit. Nyt lasi-levy on M^4xCP_2: CP_2.

      *TGD:ssä Bohrin radan analogiana ovat ”preferred ekstremaalit” M^4xCP_2:n tasolla. M^8:n tasolla ne ovat reaalipolynomien määrittelemiä algebrallisia pintoja. Ne jatketaan oktonipolynomiksi ja tämän ”imaginaariosa” joka on kvaternioarvoinen pannaan nollaa. Saadaan avaruusaikapinta.Nämä kuvataan H:n M^8-H dualiteetilla.

      *Hyvin perusteltu hypoteesi on että H:ssa saadaan minimaalipinta ekstremaalina vaikutuksen, joka on summa Maxwellin vaikutuksen analogiasta ja tilavuustermistä (kosmologinen vakio). Molemmat termit ekstremoituvat erikseen (tärkeää ja liittyy analyyttisyyteen ja konformisymmetrian yleistykseen). Myös ryhmäteoreettinen formulaatio löytyy: siinä toteutetaan ääretön määrä mittaehtojen kaltaisia ehtoja.

      >>>>>>>>>>>
      [MP] Eri avaruusaikapintojen, ei eri aikojen. Nollaenergaontologiassa kvanttitila ei ole aika=vakio snapshot vaan superpositio klassisista deterministisistä aikakehityksistä, preferred extremals, jotka ovat analogisia Bohrin radoille. Tämä seuraa yleisestä koordinaatti-invarianssista: annettuun kolmipintaan liittyy uniikki nelipinta, jolla yleiset koordinaattimuunnokset 4-D mielessä vaikuttavat. Se implikoi taas holografian: datat 3-pinnalla kiinnittävät 4-pinnan. Itse asiassa holografia osoittautuu vahvemmaksi. 2-D data kiinnittävät 4-pinnan.
      >>>>>>>>>>>

      Aloit puhua pinnoista, koska objektit ovat hiukkasen sijasta pintoja,

      [MP] Pistemäinen hiukkanen korvataan 3-pinnalla.

      %%
      mutta tässä on myös 4-pinta, joka on lopullisen kvanttiobjektin esitys eikä lehti, jossa on suljettu pinta?

      [MP] 4-pinta vastaa yleistystä pistemäisen hiukkasen 1-D radalle, 3-D objektin ”rataa”.

      %%
      Silloin tätä ns. 4D-superposition kiinnittämistä 3D tiloilla kannattaa verrata siihen etteivät Hilbertin avaruuden superpositiotkaan ole omia ylempiä systeemejään. Jos tässä taas ovat kyseessä koordinaattiavaruudet tai monistot ja alimonistot, niin niille ei ole menetelmää luoda suuremman dimension metristä tensoria pelkistä indusoiduista tensoreista, vaikka nämä kaikki tunnettaisiin esim. lehditetylle avaruudelle. Lehtien suunnan parametrin, josta tulee koordinaatti, metriikka täytyy antaa erikseen. Silti tietysti lehtien ja pienempien avaruuksien muodostus on vain matemaattista, ja jos gravitaatioteoriassa sanotaan jotain, mistä suuren avaruuden kaarevuus tulee, niin on samantekevää, onko sen lähteen muualla esitetty olevan lehdillä.

      [MP] Metriikka ja spinorikonnektio sekä 3- että 4-D pinnoilta määräytyy induktiolla, joka on TGD:n peruskäsite ja toimii myös spinoreille ja twistoreille twistoriliftissä TGD:lle. Einsteinin geometrisointi yleistyy sekä gravitaatioon että mittavuorovaikutuksiin. Variaatioperiaate määrää H:ssä avaruus aikapinnan ekstremaalinaan. Preferred extremal (Bohrin rata-) ominaisuus tulee lisäksi ja siitä jo puhuin.

      H ja H:n metriikka eivät ole dynaamisia kuten säiemallissa (spontaani kompaktifikaatio) ja määräytyvät monellakin tavalla.

      *Ensinnäkin standardimalli fiksaa H:n: H= M^4xCP_2.

      *M^8-H dualiteetti tekee saman ja dekompositiolla on lukuteoreettinen tulkinta.

      *Twistoriliftin olemassaolo fiksaa H:n. Twistoriavaruudella on oltava Kahler rakenne: M^4 (E^4 ja S^4) ovat ainoat avaruudet joiden twistoriavaruudelle tämä pätee. S^4 sulkeutuu pois koska sillä itsellään ei ole Kähler rakennetta. M^4 ja E^4 ovat ovat kompleksioidun M^8 aliavaruuksia ja M^8_c tarvitaan, mutta tämä on enemmänkin tekninen detalji.

      *”Klassisten maailmojen maailman” (WCW) täytyy sallia Kähler geometria, joka geometrisoi hermiittisen konjugoinnin. Jo looppiavaruuksille tämä geometria on uniikki vielbein konnektion olemassaolosta: vaaditaan maksimaaliset isometria. TGD:ssä tilanne on vielä vaativampi. Hypoteesi on että WCW sallii Kähler geometrian vain kun H=M^4xCP_2. Supersymplektiset symmetria Delta M^4_+xCP_2:lle, (valokartio xCP_2) olisivat nämä isometriat.

      >>>>>>>>>>>
      [MP ]M^8-H dualisuus merkitsee sitä että avaruusaika voidaan tulkinta pintana M^8:ssa tai H:ssa. M^8 kuvassa avaruusaika määräytyy polynomista rationaalikertoimin ja tämä polynomi määrää algebrallisen laajennuksen ja avaruusaikapinnan tason evoluutiohierarkiassa.

      Algebralliset laajennusten parametrit kuten dimensio n=h_eff/h_0 (h= 6h_0 seuraa empiirisesta argumentista) ja ramified primes ovat tärkeitä ja kytkentävakiot ja niiden diskreetti evoluutio laajennuksen funktiona määräytyisi näistä paramereista. On varmasti muitakin laajennusta kuvaavia lukuteoreettisia parametreja. Sironta-amplitudien lausekkeet yksinkertaistuvat, koska avaaruusaika diskretoituu uniikeiksi kognitiiviseksi esitykseksi. Tässä tulee mukaan lukuteoria, joka on tällä hetkellä voimakkaasti kehittymässä.
      >>>>>>>>>>>

      Miksi avaruusajan reaaliosien pitäisi laajentua algebrallisest?

      [MP] Ei reaaliosien vaan avaruusajan diskretoinnin, jolle pisteet ovat rationaalisia. Tulkinta on kognitiivisena esityksenä
      joka on mielekäs myös p-adisten lukukuntien laajennuksille. Esimerkiksi käyrälle x^mn+y^n=z^n rationaalinen kognitiivinen esitys on triviaali: laajennusta tarvitaan.

      Tämä diskeraatio on se mitä avaruusajasta pystytään kognitoimaan ja se myös antaa diskretoidun approksimaation sironta-amplitudeille. Evoluutiosta algebrallisen kompleksisuuden kasvuna on kyse. Avaruusaikapinta itse tulee kompleksisemmaksi.

      %%
      Jos siitä on hyötyä, niin miksei niitä laajenneta aina kompleksisiksi, missä laajennus päättyy? Monisto C^4 vastaa R^8 monistoa ja minkä tahansa pinnan funktiot saadaan C^4:n funktioista ja pinnan määritelmästä. Näin tulisi ajatella, jos R^8:n oletetaan olevan aina olemassa ja kaikki muu on sen alimonistoa, mutta tämä tarkoittaa juuri sitä, että esim. yksi p-adinen avaruus on R^2:n (R^1:n?) alimonisto ja tehty R^n -> R kuvauksella, vaikka kuvaus R -> Q_p -> R jossain muualla pitäisi niitä edes kuntana.

      [MP] Kognitio on äärellistä. Jos algebrallisten lukujen raja on koskaan saavuttamaton. Mutta parhaamme yritetään!

      %%
      Ylempi monisto ja avaruus pelkän karteesisen tulon tai yhteisen Hilbertin avaruuden asemasta vaikuttaa algebroja enemmän, siihen että R^8 -avaruudella määritellään aliavaruuden kaarevuus. Tällä sivulla lukee jossain muussa tekstiosuudessa, että CP-tapauksessa TGD:stä sillä on joka R^4 pisteessä myös CP2-avaruus, mikä voi tarkoittaa tensoritulon projektio-H:ta, missä kvanttiobjektin puhtaat tilat ovat puolijanoja ja yleisesti mikä tahansa funktionaali.

      [MP] M^8 (tai M^8_c) ja H= M^4xCP_2 ovat eri avaruuksia ja M^8-H dualiteetti sanoo sen että voidaan käyttää jompaa kumpaa. Kuvaus joka vie M^8:n pinnan M^4xCP_2:n pinnaksi vaatisi muutaman sivun tekstiä. Olennaista on, että jos M^4:n 4-D pinnan tangenttiavaruus on kvaternioninen ja sisältää preferoidun M^2:n tangenttiavaruudessaan niin tangenttiavaruus vastaa CP_2n pistettä. Siksi pisteen M^4 osa ja tangenttiavaruus voidaan kuvata pisteeksi M^4xCP_2ssa. Vastaavuus on ei-lokaali. Tämä selittää sen miksi algebralliset pinnat kuvauvat osittais-differentiaaliyhtälöiden määräämiksi minimipinnoiksi ja preferred ekstremaaleiksi.

      M^8-H dualiteetti on tavattoman voimakas niinkuin säiemallienkin dualiteetit ja tuottaa täysin odottamattomia ”insighteja” kuten sen mitä evoluutio on ja mitä valaisematon aine on.

      %%
      Vaikka () x CP2 olisi vain tällainen Hilbert-versio, johon tehdään kuvaus R^8:sta, on olemassa kaareva C^2 alimonisto, joka ei ole tavallinen aika-avaruus, ja jonka dynaaminen kaarevuus (vakio?) tuottaa aina eri kuvauksen esim. samalle objektin alkutilalle ja aikavälille. Jos taas projektioavaruus on leikkauksella muodostettu avaruus isäntäavaruudesta, niillä on molemmilla kaarevuus, ja nämä ovat R^8:n tai C^4:n alimonistoja. Muutoin, jos CP:tä käytetään vain SM-symmetrioiden vuoksi, voidaan unohtaa molemmat tavat käyttää R^8:aa ja ohittaa ongelmia, joita tulee 8D-ulotteisesta gravitaatiosta, missä kaikkien ylimääräisten ulottuvuuksien pitää kaareutua sulkemaan avaruutta lähelle R^4:ää (ilman hyvää dynaamista syytä, vaikka dynaaminen syy antaa vakion, kuten tässä aina?) mutta miten jokin muu TGD tuli koskaan toimeen, ja mitä nämä muut algebralliset laajennukset ovat verrattuna symmetrioihin, tai jos ne eivät ole samalla kaarevia?

      [MP] Muutama tarkennus.

      *H:lla ei ole mitään tekemistä Hilbert avuuksien kanssa. Se on reaalinen 8-D avaruus joka sallii kompeksisen/Kahler rakenteen (M^4:n tapauksessa sen yleistyksen).

      *Ei postuloida 8-D gravitaatiota vaan 4-D gravitaatio niin, että metriikka on indusoitu metriikka. Tämä merkitsee valtavia rajoituksia koska yleisen koordinaatti-invarianssin vuoksi neljä CP_2 koordinaattia fiksaa klassisen gravitaatiokentän ja myös standardimallin klassiset kentät neli-pinnalle. Dynamiikka on äärimmäisen yksinkertainen koska lisänä tulee vielä preferred extremal ominaisuus. Avaruusaikapinnat M^8:ssa määrävät n-asteisen polynomin n juurta! Holografia ei voisi olla enää vahvempi!

      Säieteoriassa päädyttiin pitkien seikkailujen jälkeen 10-D vai oliko se 11-D gravitaatioon tunnetuin surullisin seurauksin.

      *Kenttäteoriarajalla spinorikonnektionkomponentit eri lehdille summataan mittapotentiaaleiksi ja metriikan poikkeamille M^4 metriikasta tehdään samoin. Saadaan standardimally plus GRT. Einsteinin yhtälöt ovat jäänne Poincare invarianssista yhden avaruusaikalehden tasolla.

      *Laajennus M^8_c:ksi on välttämätön (o–> o_1+io_2 oktoniokuvassa, i kommutoi oktonio-imaginaariyksikköjen kanssa) .

      -Ensinnäkin M^8 kompleksifioitujen oktonioiden aliavaruutena ei ole suljettu oktoniotulon suhteen.

      -Toiseksi reaalipolynomien juuret ovat usein kompkleksisia x+iy. Avaruusaikapinta 4-D juurena oktonio-arvoiseksi jatketulla polynomilla on yleensä M^8_c:ssa. Reaalinena avaruusaika pinta saadaan projisoimalla M^8:aan.

      -Yksi hyöty on se, että reaalipolynomien kertolasku, summaus ja jopa jako joka tuottaa rationaalifunktioita indusoivat vastaavat operaatiot nelipinnoille. Voi sanoa, että avaruusaikapinnat ja niiden vastineet H:ssa ovat kuin lukuja: polynomien muodostama algebra on todellakin lukukunnan analogia. Myös laajennus analyyttisiin reaalifunktioihin rationaalikertoimin on kuviteltavissa.

      >>>>>>>>>>>
      [MP] Tämä on nollaenergiaontologiaan perustuvan tietoisuuden teorian ennustus. En lähde liikkeelle mistään itämaisesta filosofiasta, jonka yrittäisin reprodusoida. Tämä kahtiajako vastaa sitä, etttä kvanttihyppyjen sarjassa CD:n passiiviseen reunaan liittyvä tila ei muutu: Zeno efektin analogia. Aktiiviseen reunaan liittyvä tila muuttuu jokaisessa unitaarisen evoluution vastineessa jota seuraa ”heikko” mittaus. Tämä vastaa aistisyötettä. Iso (”tavallinen”) kvanttihyppy merkitsee ”itsen” kuolemaa ja reinkarnaatiota vastakkaisessa ajan suunnassa. Nyt aktiivisesta CD:n reunasta tulee passiivinen ja päinvastoin. Tälle antaa suoraa tukea Minev et al havainnot kuten myös Libet.
      [MP] ”Itsen kehitys” on analogia Zeno efektille. Nolla-energiaontologian tilat ovat pareja tavallisista 3-D tiloistan CD:n reunoilla. CD:n passiivisella reunalla tila ei muutu – Zeno efekti. Aktiivisella reunalla, jossa tapahtuu heikkoja mittauksia se muuttuu ja se vastaa ”itsen” aistihavaintoja.
      >>>>>>>>>>>

      Rinnastetaanko vain passiivinen reuna ns. gravitonien tai gravitaatiokentän aiheuttamaan tilan romahtamiseen?

      |MP] Ei. Passivisella reunalla ei tapahdu mitään. Zeno efekti. Aktiivisella reunalla SSFR:t (small state function reductions) tuottavat ”itsen” normaalitajunnan heikkoina mittauksia: aistihavaintoja ja niiden tuottamia ajatuksia. Ihan tavallista elämää, ei mitään dramaattista kuten BSFRiä, joissa otetaan ja kuollaan ja reinkarnoidutaan vastakkaisessa ajan suunnassa.

      %%
      Kun esim. Bohrin ratkaisussa (n=0) elektronin paikka ei koe sellaista vuorovaikutusta, että radan pitäisi muuttua pisteeksi, sen ja gravitaation systeemi on varmasti joko erilainen kuin on sähkömagneettisesti mitatulla elektronilla tai sitten gravitaatio romahduttaa hyvin pienen joukon systeemeistä eikä välttämättä yhtään paikkamuuttujaa.

      [MP] Kyllähän Bohrin rata joka on piste, vastaa avaruusajassa 1-D rataa niinkuin muutkin radat.

      %%
      Kaikilta näiltä teorioilta voi tällöin kysyä, tiedetäänkö näistä systeemeistä tällöin jotain, ja TGD:ltä kysyn, onko TGD:n passivinen reuna varmasti sellainen, että siellä tämä vaikutus esiintyy muuttei muualla ja onko sen määrittelemä tila ja sen herkkyys samanlainen kuin muissa teorioissa. Ensin esimerkiksi se, että päteekö romahdus kaikille tiloille ja suureille (tämä voi johtaa myös epäkommutatiivisuuteen ja planckin rajaan, mutta gravitaation/avaruuden rooli ja siihen kytkeytyminen kiinnostaa enemmän)?

      [MP] En taida ymmärtää kysymystä. Niinkuin totesin passiviinen reuna on todella passiivinen. Ei tapahdu mitään kolmipinnoille eikä passivisen reunan 3-D kvanttitiloille. Vain sieltä alkavien nelipintojen tangenttiavaruudet muuttuisivat. Vastaisiko tämä tunnetta siitä että jotenkin olen edelleen se sama ihminen kuin lapsena. Aktiivisuus on aktiivisella CD:n reunalla.

      %%
      Moniin kenttäteorioiden esim. Feynman-sovelluksiin verrattuna pelkkä valokartion pitäminen ainoana kausaalisena osana on liian rajallista, eikä yleiselle objektille, jonka paikka ja liikemäärä ovat alussa kvanttitilassa, voida antaa yhtä valokartiota. Vaikka voitaisiin, täytyisi ainakin integroida tapaukset, joissa mikä tahansa muu yhdistetyn systeemin objekti esiintyy poissa massakuorelta ja tässä tapauksessa sellaiset alkuobjektit, joiden paikan jne. keskiarvot esiintyvät ensimmäisen objektin valokartion ulkopuolelta.

      [MP] Kausaalisuuden määrittelee klassinen variaatio-peritaate pinnoille ja indusoiduille spinorikentille. Kausaalitimantti tuo mukaan reunaehdot ja minimaalipintojen yhtälö on ei-lineaarine yleistys massattomalle aaltoyhtälölle.

      Fermionisessa sektorissa fermionipropagaattorien ominaisuudet M^4xCP_2ssa koodaavat kausaalisuuden kenttäteorioiden tapaan. Indusointi second quantized spinoreille H:ssä kehitettynä massattoman (8-D mielessä) Diracin moodien avulla määrää propagaattorin fermioneille avaruusaikapinnallakin. Ja M^4 impulssit tulevat sirontaamplitudeihin.

      Massakuorella olo vastaisi fermioneille sitä että 4-D massa = H:n Diracin yhtälön moodin määräämä. Dirac propagaattori Hssa voidaan tulkita niin että mukana ovat virtuaaliset massakuorelta poissa olevat tilat. Massan neliöön fysikaaliselle tilalle kontribuoivat vibraatio ekskitaatio koska kyseessä on kolmipinta. Ei pistemäinen hiukkanen.

      %%
      Minev ei palauta tilaa kääntämällä ajan suuntaa. Hänellä saattaa olla kaikissa liikeyhtälöissä ajankääntö-symmetria, mutta tämä on voinut olla sivuseikka eikä ole se, mikä on johtanut eri asetuksiin. Kun Minev nimittäin kääntää tilan evoluution, hän vaihtaa koko evoluutiooperaattorin käyttämällä uusia asetuksia ulkoisille kentille. Tällöin systeemi A, joka pysäytetään ei sisällä mitään samoja todennäköisyyksiä lopputilalle kuin romahtamista jatkava systeemi B, vaikka teoriassa molempien vektoritilojen kannat ovat samat (A ja B tehdään aina eri hiukkasille, jolloin maailman Hilbertin avaruudessa kannat eivät ole samat). A:ta ja B:tä kuitenkin verrataan keskenään ja B:n tiloja pidetään jonain A:n muuna vaihtoehtotulevaisuutena, vaikka näin ei ole.

      Entä jos Minev antaa ison kuolemahypyn tapahtua, ja tämä hyppy kestää kauan, ja timantin alareunasta vain osio, joka on t > dt yläpuolella, on aktiivinen tästä eteenpäin, niin mitä gravitatiivinen vaikutus näille reunan osille sitten tekee (Minev on mm. gravitaatio, mutta jatkaako hän)? Entä jos Minev ei anna hypyn tapahtua, mutta saavuttaa vain pienemmät todennäköisyyserot kuin 0.8-0.2 (hänellä alkutila) kuvassa 4b, niin eikö reunan aktiivisuuskin voi olla superpositio-tilassa?

      [MP] Minev ei oleta että ajankääntö tapahtuisi. Hänen lähestymistapansa on aivan toinen ja käsittääkseni klassinen aikakehitys kvanttihypyn lopputilaan vain postuloidaan ja syy siihen miksi se alkaa täytyisi jonkinlainen kriittisyys jollain hetkellä joka sitten valitsisi yhden lopputilan. Tällaista kriittisyyttä on vaikea ymmärtää.

      Evoluution peruuttaminen, joka sekin havaintaan tietyillä häiriöillä olisi TGD:ssä näennäistä. Indusoidaan vastakkainen kvanttihyppy ja paltaan alkuperäiseen ajan suuntaan. Ikäänkuin mitään ei sitten tapahtuisikaan.

      >>>>>>>>>>>
      [MP] Ei ihan näin. Kytkentävakioita ei imetä h:n.
      >>>>>>>>>>>

      %%
      Jos h kasvaa ja kytkentävakiot pysyvät vakiona, tiedetään että vapaan hiukkasen paikka-aalto on leveämpi. Aaltopaketit leviävät ajassa nopeammin suuremmalla h:lla. Tämä vapaan hiukkasen paikan todennäköisyyden diffuusio on tällä sivulla:
      https://quantummechanics.ucsd.edu/ph130a/130_notes/node83.html

      Systeemien ominaisenergiat kasvavat, ja varmaan esim. hiukkanen laatikossa ei toteutuisi leveälle aallolle ilman vastaavaa suurempaa potentiaalia.

      Pelkästään h:sta riippuva käytös on mahdollisesti peräisin vain ensimmäisiin polkuintegraali-metodeihin johtaneesta kvanttivaikutuksesta ja siitä johdetusta mekaniikasta, joka perustui silloin klassisen Lagrangen funktion vaikutukseen ja sen poikkeuksiin, jolloin poikkeusten häviämistä on voitu pitää klassisena rajana. Ja näitä häviämisten sijaan useammin tutkittiin vain h:n nollan lähestymistä ternissä e^(iS/h).
      https://arxiv.org/abs/1201.0150

      [MP] Kvanttipituudet skaalaavat tyypillisesti kuten hbar. Atomeille kuten hbar^2. Systeemin kvanttikoko joka korreloi vastaavan avaruusaikapinnan koon kanssa kasvaa. Esimerkiksi diffraktiossa spottien koko kasvaa.

      Ei standardi h:t olisivat realisoituneet valaisemattoman aineen tasolla magneetisella keholla (MB). MB kontrolloisi tavallista ainetta fiksumpana koska h_eff =nh_0 toimii IQna (algebrallisen kompleksisuuden mittana) koska se vastaa polynomin astetta. Jos h_eff kasvaa niin MB:n kontrolloiman aluee koko kasvaa ja samoin kontrollin aika-skaala. Tämä saattaisi tehdä mahdolliseksi testata.

      Mielenkiintoinen efekti on esimerkiksi alfan skaalautuminen kun h_eff. Yksi motivaatio faasitransitiolle joka kasvattaa h_eff:ä on se, että muutoin häiriöteoria ei konvergoi. Universumi on pohjimmiltaan ja kaikesta huolimatt teoreetikko-ystävällinen ja järjestänyt valaisemattoman aineen h_eff=nh_0 faaseina säästäkseen teoreetikot ei-perturbatiivisen teorioidan tuottamalta loputtomalta turhautumiselta.

      %%
      Artikkelissa muistutetaan kovasti, että raja vastaa silloinkin probabilistisia liikeyhtälöitä (ehkä rho:kin on diagonaalinen), mutta siinä on yksi tapaus (9) pienelle joukolle kvanttitiloja, jossa näkyy h:n osuus verrattuna osuteen ilman h:ta. Myös lähtökohdan ongelmia selvitetään sivulla 9-10.

      https://en.wikipedia.org/wiki/Classical_limit

      [MP] Idea kvanttidynamiikan tulemisesta klassiseksi tulee tarpeettomaksi ZEOssa. Kvanttihyppy näyttää aina klassiselta havaitsijalle jonka ajan suunta on toinen- siis standardi. Tietysti kvanttitilojen klassisuudesta voidaan puhua aivan kuten normaalisti.

      %%
      Sanoo periaattessa, että kvanttioskillaattori (jolle on annettu klassinen energia) muuttuu klassiseksi, joissakin korkeampien energioiden ominaistiloissa. Jos suuremmalla energialla absoluuttisesti olisi oikeasti merkitystä systeemin kvanttiluonteen pitämiseksi, tätä kompensoi se, että energiat (ei klassisen vaan kvanttitilan) ovat jo suurempia ja kvanttiluku n määrittäisi klassisen rajan ilman h:ta. Energiaa esiintyy myös yksittäisen ison h:n muodostamassa suuressa irrallisessa populaatiossa jatkuvasti, koska suuret energiavälit muodostavat korkeampienergisiä bosoneja, joita koko systeemi joutuu vaihtamaan. Pelkkä bosonin emittointi johtaa klassisuuden kasvamiseen ja voi olla verrannollinen energiaan ja muutos on paikkatilalle lopulta itsenäinen h:sta.

      Jos tuo yksi malli kuitenkin unohdetaan eikä pidetä yksittäisen systeemin klassisuuden muutosta edes mahdollisena täydellisessä vakuumissa, niin kvanttiluonto häviää vain vuorovaikutuksissa, mitä voi tulla objektille enemmän johtuen sen ja muiden objektien paikkatilojen kasvavasta päällekkäisyydestä, mitä objektit varmaan kompensoivat olemalla harvemmassa pelkkien törmäysten avulla (aine on hetken aikaa klassisempaa) tai jopa mudostamalla avaruuteen suuremman osan bosonista ainetta ja laajentamalla sitäkin nopemmin kuin pienellä h:lla (sanoit tosin jotain massan laskemisesta ja ehkä uusiksi?). Ja vuorovaikutukset mitä näin tulee pysyvät samoina tai hidastuvat.

      Esim. Minevin artikkelissa ei sanota, mutta h esiintyy /Omegoiden nimittäjissä ja tavallisesti myös radiaatio-kertoimissa (spontaani emissio), nimittäjissä, ja tilat, romahtamiset ja pakittamiset ovat esim. tanh(x/h), eli suuremman h:n tila kehittyy hitaammin (”dekoherentissa ajossa” h:ta ei ole). Kaikissa muissakin unitaarisissa kehityksissä tai melkein kaikille energioille, jotka ovat h-verrannollisia, aikakehitysopraattori on h:sta vapaa, ja h ei vaikuta hetken t tilojen todennäköisyyksiin. Objektien toisiinsa vaikuttaminen ei siis muutu eikä edes tajuus oman esimerkkini a-oskilloinneissa muutu. Se tarkoittaa ettei kietoutuminenkaan muutu niissä samaan perustuvissa tapauksissa, mitä kietoutumisen unitaarisesta mudostumisesta tiedetään.

      [MP] Totta on että esim. perturbatiivisessa tapauksessa h ei näy alimmassa kertaluvussa sirontavaikutusaloissa. Vety-atomi on taas ei-perturbatiivinen systeemi (sidosenergia skaala E propto 1/hbar^2).

      h_eff:n muutos TGD:ssä olisi karkeimmalla tasolla kvanttiskaalojen skaalaus.

      Mukana on kuitenkin paljon enemmän. Avaruusaikapintojen määräävän polynomin aste n=h_eff/h_ 0 kasvaa. Juurien määrä kasvaa samassa suhteessa kuin pinnan koko ja niiden määräämä algebrallinen laajennus kasvaa dimensioltaan. Kognitiiviset esitykset pisteinä joille avaruusajan pisteen koordinaatit ovat tässä laajennuksessa kasvava kooltaan. Esitys tulee tarkemmaksi. Karkea hatusta heitetty arvaus on, että pisteiden määrä tilavuusyksikköä kohden säilyy.

      Tykkää

Jätä kommentti

This site uses Akismet to reduce spam. Learn how your comment data is processed.